Nursing 1 final Part 2

Pataasin ang iyong marka sa homework at exams ngayon gamit ang Quizwiz!

Which of the following is an example of theoretical knowledge? 1) A nurse uses sterile technique to catheterize a patient. 2) Room air has an oxygen concentration of approximately 21%. 3) Glucose-monitoring machines should be calibrated daily. 4) An irregular apical heart rate should be compared with the radial pulse.

2) Room air has an oxygen concentration of approximately 21%. Rationale: Theoretical knowledge consists of research findings, facts, principles, and theories. The oxygen concentration of room air is a scientific fact. The others are examples of practical knowledge—what to do and how to do it.

Crepitus

Creaking of the joint

Ankylosis

Fusing of the joints

Scoliosis

lateral curvature of the spine

The nurse intends to identify nursing diagnoses for a patient. She can best do this by using a data-collection form organized according to (select all that apply): 1) A body systems model 2) A head-to-toe framework 3) Maslow's Hierarchy of Needs 4) Gordon's functional health patterns

1) A body systems model 4) Gordon's functional health patterns Rationale: Nursing models produce a holistic database that is useful in identifying nursing rather than medical diagnoses. Body systems and head-to-toe are not nursing models, and they are not holistic; they focus on identifying physiological needs or disease. Maslow's Hierarchy of Needs is not a nursing model, but it is holistic, so it is acceptable for identifying nursing diagnoses. Gordon's functional health patterns constitute a nursing model.

Using a "patient preferences" framework, which of the following nursing diagnoses would probably have the highest priority for a patient who fractured his leg yesterday and now is wearing a cast? 1) Acute Pain 2) Disturbed Body Image 3) Ineffective Peripheral Perfusion 4) Impaired Physical Mobility

1) Acute Pain Rationale: Using a patient preferences framework, highest priority would be given to whatever the patient thinks is most important, which most often would be pain. The nurse would, of course, be concerned about mobility, body image, and tissue perfusion. If using another framework (e.g., the potential for permanent injury), the nurse might be most concerned about Ineffective Peripheral Perfusion.

In which of the following ways do collaborative problems differ from nursing diagnoses? Choose all correct answers. 1) All patients who have a certain disease are at risk for developing the same problem. 2) Collaborative problems are always potential problems. 3) The complications can be prevented with nursing interventions alone. 4) The problem statement does not need to be approved by a medical provider.

1) All patients who have a certain disease are at risk for developing the same problem. 2) Collaborative problems are always potential problems. Rationale: All patients who have a particular disease are at risk for developing the same complications (or collaborative problem). Patients with the same disease do not necessarily have the same nursing diagnoses (e.g., after a heart attack, one patient may have Anxiety; another may not). Therefore, collaborative problems differ from nursing diagnoses in this respect. It is true that collaborative problems are always potential problems; nursing diagnoses may be but are not always potential problems. Therefore, collaborative problems and nursing diagnoses differ in this respect. If complications can be prevented with nursing interventions alone, it is a nursing diagnosis, not a collaborative problem. This statement is not true of collaborative problems, so that cannot be a characteristic that makes them different from nursing diagnoses. Neither collaborative problems nor nursing diagnoses need to be approved by a medical problem. The two types of problem are the same in this respect, not different.

What assessment findings might the nurse expect to see in a patient experiencing hypoxia? Select all that apply. 1) Altered level of consciousness 2) Peripheral pitting edema 3) Cyanosis of skin and mucous membranes 4) Weak or absent peripheral pulses 5) Jaundiced sclera

1) Altered level of consciousness 3) Cyanosis of skin and mucous membranes 4) Weak or absent peripheral pulses Rationale: Hypoxia leads to decreased oxygenation of organs and tissues. To determine adequacy of tissue oxygenation, you must assess both circulation and tissue/organ function. An altered level of consciousness may result from hypoxic central nervous system tissue. Poor peripheral circulation is characterized by weak or absent pulses; pale, ashen, or cyanotic skin and mucous membranes; and cool skin temperature. Peripheral edema does not result from hypoxia.

Critical thinking and the nursing process have which of the following in common? Both: 1) Are important to use in nursing practice 2) Use an ordered series of steps 3) Are patient-specific processes 4) Were developed specifically for nursing

1) Are important to use in nursing practice Rationale: Nurses make many decisions: Some require using the nursing process, whereas others are not client related but require critical thinking. The nursing process has specific steps; critical thinking does not. Neither is linear. Critical thinking applies to any discipline.

As the nurse is inserting a urinary catheter she observes blood in the urine returned in the tubing. Which two nursing process phases does this demonstrate? 1. Assessment 2. Diagnosis 3. Implementation 4. Planning: Interventions

1) Assessment 2) Implementation Rationale: Assessment occurs when the nurse is gathering data (i.e., observing blood in the urine). Implementation occurs when the nurse is performing a nursing action (i.e., inserting a urinary catheter). Diagnosis occurs when the nurse analyzes the data and states what they mean—which she did not do in this situation. Planning: Interventions occurs when the nurse is considering the patient diagnoses and goals and deciding what actions need to be taken.

A young adult client is in the hospital after a motor vehicle accident. He is in a body cast and essentially immobile, but he is otherwise stable. Which activity or activities can the nurse delegate to a nursing assistant? Choose all that apply. 1) Bathing the client 2) Taking the client's vital signs 3) Choosing nursing interventions 4) Prioritizing interventions

1) Bathing the client 2) Taking the client's vital signs Rationale: The registered nurse is responsible for choosing and prioritizing interventions. However, because the client is stable, the nurse can delegate physical care to a nursing assistant as long as the nursing assistant has the required level of competence and the activity does not involve a high level of decision making. The nurse can delegate a defined activity in a particular situation, but cannot delegate nursing care decisions.

How can the nurse help promote client participation and adherence to a treatment regimen? 1) Be certain the care plan considers cultural and spiritual needs. 2) Explain that the treatments were prescribed by a physician. 3) Acknowledge that treatment is difficult, but that other patients do manage it. 4) Provide detailed written instructions about the client's disease process.

1) Be certain the care plan considers cultural and spiritual needs Rationale: Altering interventions and treatments to reflect client preferences and beliefs helps to promote adherence. Knowing that a physician prescribed the treatment regimen is not likely to motivate the patient. It is important to acknowledge that the treatment regimen is difficult, inconvenient, and so forth. However, it could be discouraging to point out that others can manage it. Although it is important to provide written instructions about the treatments, they must be simple, clear, and specific. Detailed instructions about the disease process are not necessary, nor likely even helpful in promoting adherence.

Your client is a healthy, older adult who has comes to the health clinic because she reports not feeling like herself. When you are gathering data in your client's health history, she tells you that she is feeling more fatigue when walking up stairs and doing her normal household activities. What are normal physiological changes in the cardiovascular system that occur with aging? Select all that apply. 1) Cardiac contractile strength is reduced. 2) Heart valves become more rigid. 3) Peripheral vessels lose elasticity. 4) Heart responds to increased oxygen demands.

1) Cardiac contractile strength is reduced. 2) Heart valves become more rigid. 3) Peripheral vessels lose elasticity. Rationale: Cardiac efficiency gradually declines as the heart muscle loses contractile strength and heart valves become thicker and more rigid. The peripheral vessels become less elastic, which creates more resistance to ejection of blood from the heart. As a result of these changes, the heart becomes less able to respond to increased oxygen demands, and it needs longer recovery times after responding.

A patient is obese and complains of fatigue with activity. The nurse is planning goals for this patient. Which of the following NOC outcome(s) relate(s) directly to the above nursing diagnosis; that is, which outcome(s), if achieved, would demonstrate resolution of her problem? Select all that apply. 1) Endurance 2) Activity Tolerance 3) Active Joint Movement 4) Mobility Level

1) Endurance 2) Activity Tolerance Rationale: Both Endurance and Activity Tolerance, if demonstrated by this patient, would show that her problem (Activity Intolerance) is improving. Active Joint Movement and Mobility Level relate to part of the etiology of her problem, which is lack of activity. Even if she demonstrates joint movement and other mobility, this would not necessarily indicate that she is tolerating the activity well.

The nurse is caring for an older adult woman who was admitted 3 days ago following a cerebrovascular accident. She has had trouble swallowing and has been placed on aspiration precautions. Care of this patient will include the following. Select all that apply. 1) Ensure she is sitting upright or with the head of the bed elevated to eat and drink. 2) Break or crush her pills (if appropriate) before administration. 3) Provide only thin, clear liquids. 4) Keep suction setup available at all times.

1) Ensure she is sitting upright or with the head of the bed elevated to eat and drink. 2) Break or crush her pills (if appropriate) before administration. 4) Keep suction setup available at all times. Rationale: Aspiration is a risk for patients with a decreased level of consciousness, diminished gag or cough reflex, or difficulty with swallowing. You should keep suction setup available for routine and emergency use. Keeping the head of the bed elevated will also help to prevent aspiration. Breaking or crushing pills will make it easier for her to swallow her medications. Thin, clear liquids are more likely to be aspirated than are thickened liquids, and there is no indication for clear liquids.

A facility is using ___________ nursing care when scientifically sound research data are used to make nursing care decisions. 1) Evidence-based 2) Standardized 3) Individualized 4) Theoretical

1) Evidence-based Rationale: Evidence-based nursing care uses solid, scientific data to make decisions about nursing practice.

The nurse works rotating 12-hour day and night shifts. She wakes up after about 3 or 4 hours and has trouble going back to sleep. Feeling unrefreshed, nauseated, irritable, and headachy, she consults another nurse for advice on sleep hygiene. What is the best advice for avoiding sleep deprivation? 1) Follow a regular routine h.s. and after awakening. 2) Eat a small, carbohydrate snack shortly before bed. 3) Take melatonin consistently before going to sleep. 4) Take a 30- to 60-minute nap during the afternoon.

1) Follow a regular routine h.s. and after awakening. Rationale: Follow a regular routine for bedtime and morning awakenings. This helps the body to regulate a natural circadian cycle. This means you should go to bed each night at the same time, even on days you are off work. Avoid eating carbohydrates (e.g., crackers, cereal, or bread) before bed; they boost blood glucose levels, so a few hours later, the rapid drop in sugar will wake you. The effectiveness of melatonin as a sleep aid remains controversial. It is not shown to be effective for shift-related sleep deprivation or jet lag. Although approved by the Food and Drug Administration (FDA), melatonin is unregulated and may vary in strength and purity across manufacturers. Avoid naps during the day, unless you are an older adult who takes short "power naps." Daytime napping can lead to nighttime insomnia.

What is the most basic reason that self-knowledge is important for nurses? Because it helps the nurse to: 1) Identify personal biases that may affect his thinking and actions. 2) Identify the most effective interventions for a patient. 3) Communicate more efficiently with colleagues, patients, and families. 4) Learn and remember new procedures and techniques.

1) Identify personal biases that may affect his thinking and actions Rationale: The most basic reason is that self-knowledge directly affects the nurse's thinking and the actions he chooses. Indirectly, thinking is involved in identifying effective interventions, communicating, and learning procedures. However, because identifying personal biases affects all the other nursing actions, it is the most basic reason.

How are critical-thinking skills and critical-thinking attitudes similar? Both are: 1) Influences on the nurse's problem-solving and decision making 2) Like feelings rather than cognitive activities 3) Cognitive activities rather than feelings 4) Applicable in all aspects of a person's life

1) Influences on the nurse's problem-solving and decision making Rationale: Cognitive skills are used in complex thinking processes, such as problem-solving and decision making. Critical-thinking attitudes determine how a person uses her cognitive skills. Critical-thinking attitudes are traits of the mind, such as independent thinking, intellectual curiosity, intellectual humility, and fair-mindedness, to name a few. Critical-thinking skills refer to the cognitive activities used in complex thinking processes. A few examples of these skills involve recognizing the need for more information, recognizing gaps in one's own knowledge, and separating relevant from irrelevant data. Critical thinking, which consists of intellectual skills and attitudes, can be used in all aspects of life.

Which of the following functions both as a care plan and a documentation form? Choose all correct answers. 1) Integrated plan of care (IPOC) 2) Critical pathway 3) Individualized patient care plan 4) Standardized (model) care plan

1) Integrated plan of care (IPOC) 2) Critical pathway Rationale: IPOCs are standardized plans that function as care plans as well as documentation forms. Many critical pathways are designed as IPOCs, and may function as documentation forms; however, not all critical pathways are IPOCs. Individualized care plans are not designed to be used as documentation forms. Although some standardized care plans provide writing space in which to individualize nursing orders and patient goals, they do not provide a place to document care and patient responses to care.

In diagnostic reasoning, which of the following does the nurse usually do first? 1) Interpret patient data. 2) Draw conclusions about health status. 3) Verify problems with the patient. 4) Prioritize health problems.

1) Interpret patient data. Rationale: The diagnostic reasoning process allows the nurse to make sense of patient data for planning and providing quality nursing care. The broad steps in diagnostic reasoning occur generally in this order: Analyze and interpret data, draw conclusions about health status, verify problems with the patient, and prioritize problems. Of course, the steps overlap, and the nurse may move back and forth between them.

What are prevailing characteristics of narcolepsy? Select all that apply. 1) Involuntary 2) Cataplexy 3) Hallucinations 4) Temporary paralysis

1) Involuntary 3) Hallucinations Rationale: The person with narcolepsy experiences a sudden, uncontrollable urge to sleep lasting from seconds to minutes, even though the person sleeps well at night. The person cannot avoid the "sleep attacks" but awakens easily. Narcolepsy is characterized by involuntary episodes of sleepiness, slurred speech, slackening of the facial muscles, a feeling of impending weakness of the knees, paralysis, and hallucinations. Some have other symptoms, such as cataplexy, a sudden loss of muscle tone usually triggered by an emotional event (e.g., laughter, surprise, or anger), but most only have hypersomnia.

Using the mnemonic H E L P (from Volume 1 of your textbook), which of the following correctly illustrates the letter E—Equipment and Environment? 1) Is the oxygen running? 2) Who else is in the room with the patient? 3) Does the patient have any questions? 4) Take a thorough look at the patient.

1) Is the oxygen running? Rationale: H: (Help): Look for signs of patient distress E: (Equipment and Environment): Check for safety hazards and equipment operation. L: (Look more closely): Look more closely at the patient for cues that care may need to be given. P: (People): Who else is in the room?

When writing an individualized patient care plan, which of the following should the nurse do first? 1) Transcribe medical orders to the appropriate documents. 2) Individualize standardized plans as needed. 3) Write basic care needs on the Kardex or in special sections of the care plan. 4) Make a working problem list with problems in priority order.

1) Make a working problem list with problems in priority order. Rationale: Writing an individualized plan of care follows in natural sequence from the assessment and diagnosis phases of the nursing process. In those phases, the nurse has already developed and prioritized a list of the patient's problems and strengths. The nurse must start with this list and then decide which problems can be managed with standardized plans or critical pathways. Then the nurse individualizes the standardized plan(s) as needed. After that, the nurse enters the medical orders in the appropriate documents and writes ADLs and basic care needs in special sections of the care plan or Kardex. Finally, the nurse develops individualized (nursing diagnosis) care plans to address problems not covered by the standardized documents.

An 80-year-old resident in a long-term-care facility comes to the emergency department with dehydration. The nurse writes a diagnosis of Deficient Fluid Volume related to excessive fluid loss. An individualized nursing goal identified for this client is "The client will maintain urine output of at least 30 mL/hour." Which nursing interventions would directly help achieve or evaluate the stated goal? Choose all that are correct. 1) Measure and record urine output every hour; report an output of less than 30 mL/hour. 2) Monitor skin turgor and moistness of mucous membranes every shift. 3) Administer intravenous (IV) fluids as prescribed. 4) Keep oral fluids within the patient's reach and encourage the patient to drink.

1) Measure and record urine output every hour; report an output of less than 30 mL/hour. 3) Administer intravenous (IV) fluids as prescribed. 4) Keep oral fluids within the patient's reach and encourage the patient to drink. Rationale: Measuring and recording urine output allow for direct evaluation of the goal "urine output 30 mL/hour." Administering IV fluids adds fluid to correct dehydration, improves blood flow through the kidneys, and increases urine production. Intake of oral fluids has the same effect. Monitoring skin turgor and monitoring mucous membranes are ways to assess for dehydration, but that intervention does not directly apply to the goal of maintaining urine output. It is aimed at the etiology of this nursing diagnosis, rather than the problem.

What is the hormone that promotes sleep? 1) Melatonin 2) L-tryptophan 3) Progesterone 4) Oxytocin

1) Melatonin Rationale: The levels of melatonin, which is the natural hormone that promotes sleep, decline in the latter decades of life. It is produced at night by the pineal gland in the brain.

Which of the following is true of the Nursing Interventions Classification (NIC)? Select all that apply. 1) NIC interventions can be used in all specialty areas of nursing practice. 2) The American Nurses Association (ANA) has approved it for use. 3) It is used mainly by home health nurses. 4) It is designed primarily for use in hospitals.

1) NIC interventions can be used in all specialty areas of nursing practice. 2) The American Nurses Association (ANA) has approved it for use. Rationale: The NIC is on the ANA's list of approved standardized languages. NIC interventions were designed for and are applicable in home, community, and acute care settings. The taxonomy designed specifically for home health is the Clinical Care Classification.

Respiratory function involves, in addition to the respiratory system, which three body systems? Select all that apply. 1) Neurological 2) Endocrine 3) Cardiovascular 4) Musculoskeletal

1) Neurological 3) Cardiovascular 4) Musculoskeletal Rationale: The musculoskeletal and neurological systems regulate the movement of air into and out of the respiratory system. The cardiovascular system transports oxygen and carbon dioxide, which are exchanged in the lungs.

A patient with high blood pressure receives a daily oral medication to control his blood pressure. However, he has been vomiting for the past 24 hours. The nurse knows that oral medications are absorbed in the gastrointestinal tract, and that without the drug, the patient's blood pressure may become dangerously high. So the nurse notifies the primary provider to see whether the drug can be given by another route (e.g., intravenously or rectally). The nurse's thinking and actions illustrate which of the following? 1) Nursing is an applied discipline. 2) Nursing uses knowledge from other fields. 3) Nursing is fast paced. 4) Nursing requires ethical knowledge.

1) Nursing is an applied discipline 2) Nursing uses knowledge from other fields Rationale: Nursing is an applied discipline—The nurse not only knew facts about blood pressure, gastrointestinal tract motility, blood pressure drugs, and vomiting; he also followed through with actions—he did not merely memorize and regurgitate the facts. Nursing uses knowledge from other fields—The nurse used knowledge from physiology (about high blood pressure and about the relationship of gastrointestinal motility to vomiting). The nurse used knowledge from pharmacology regarding the absorption of oral medications, as well as about the desired effects of the blood pressure medication. Nursing is certainly fast paced and it requires ethical knowledge. However, there is nothing in this scenario to illustrate those two characteristics of nursing. There is not enough information to assume that there was any immediate urgency in the situation that would require the nurse to rush, nor is there clearly any ethical question that arises from the situation.

Which standardized intervention vocabulary was designed specifically for community health nurses? 1) Omaha System 2) Clinical Care Classification 3) Nursing Interventions Classification 4) International Classification for Nursing Practice

1) Omaha System Rationale: The Omaha System was designed specifically for community health nurses to use in caring for individuals, families, community groups, or entire communities. The Clinical Care Classification was developed for home healthcare. The Nursing Interventions Classification system is applicable in all settings, including home health and community nursing. The International Classification for Nursing Practice system was designed to describe nursing practice of individuals, families, and communities worldwide.

A patient underwent surgery 3 days ago for colorectal cancer. The patient's critical pathway states that he should participate in a teaching session with the wound ostomy nurse to learn colostomy self-care. The patient appears depressed and refuses to look at the colostomy or even make eye contact. How should the nurse proceed? 1) Postpone the teaching session until the patient is more receptive. 2) Follow the critical pathway for patient teaching. 3) Administer a prescribed antidepressant and notify the healthcare provider. 4) Explain to the patient the importance of skin care around the ostomy site.

1) Postpone the teaching session until the patient is more receptive. Rationale: A depressed affect and poor eye contact likely indicate the client is having difficulty coping with the new colostomy. At this time, the client would not be physically and psychologically ready to obtain the most benefit from teaching pertaining to ostomy care. Therefore, the nurse should postpone the teaching session for this client until the client is receptive to receiving the information. The nurse should not perform the teaching session simply because the critical pathway indicates it is appropriate. Simply administering an antidepressant does not address the client's readiness to participate in a teaching session and ultimately self-care of the ostomy. The nurse should encourage the client to verbalize his feelings. Client education is not effective unless the client is receptive to the information. Readiness to learn is important. Proceeding with teaching when the client is struggling with coping is not sensitive to the client's individual needs.

What is the rationale for wrapping petroleum gauze around a chest tube insertion site? 1) Prevents air from leaking around the site 2) Prevents infection at the insertion site 3) Absorbs drainage from the insertion site 4) Protects the tube from becoming dislodged

1) Prevents air from leaking around the site Rationale: Petroleum gauze creates a seal around the insertion site. Collapse of the lung can occur if there is a leak around the insertion site that causes loss of negative pressure within the system. Air leaks are one common cause of loss of negative pressure.

Which of the following nursing diagnoses is written in correct format? Assume the facts are correct in all of them. 1) Readiness for Enhanced Nutrition 2) Pain related to stating, "On a scale of 1 to 5, it's a 5." 3) Impaired Mobility related to pain A.M.B. hip fracture 4) Risk for Infection related to compromised immunity A.M.B. fever

1) Readiness for Enhanced Nutrition Rationale: Wellness diagnoses (e.g., Readiness for Enhanced Nutrition) are usually one-part statements. A pain ranking of 5 is a symptom of pain, not an etiology, so it should be preceded by "A.M.B." or as "manifested by." Hip fracture is a medical diagnosis that is causing an etiology of pain; therefore, it should be preceded by "secondary to." Risk diagnoses do not have symptoms, so it is not correct to put anything after "A.M.B."

The nurse is caring for an adolescent with a history of asthma. The patient is currently being treated for acute bronchitis. A thorough nursing assessment of the patient's pulmonary status will include. Select all that apply. 1) Skin color 2) Temperature 3) Auscultation of breath sounds 4) Testing of cough reflex 5) Chest x-ray

1) Skin color 2) Temperature 3) Auscultation of breath sounds Rationale: Auscultation of the lungs will detect any adventitious breath sounds (e.g., wheezing) that may be present with asthma and bronchitis. Evaluation of skin color and temperature are indirect methods to assess tissue oxygenation. A chest x-ray requires a physician order and is not a part of the nursing assessment. The cough reflex should be assessed in clients with decreased levels of consciousness.

The nurse is recording assessment data. She writes, "The patient seems worried about his surgery. Other than that, he had a good night." Which errors did the nurse make? Select all that apply. 1) Used a vague generality 2) Did not use the patient's exact words 3) Used a "waffle" word (e.g., seems) 4) Recorded an inference rather than a cue

1) Used a vague generality 3) Used a "waffle" word (e.g., seems) Rationale: The nurse recorded a vague generality: "he had a good night." The nurse did not use the patient's exact words, but she did not quote the patient at all, so that is not one of her errors. The nurse used the "waffle" word "seems" instead of documenting what the patient said or did to lead her to that conclusion. The nurse recorded these inferences: "worried" and "had a good night."

Which is true of unit standards of care? Unit standards of care are (select all that apply): 1) Written for a specific medical diagnosis or treatments 2) Organized according to nursing diagnoses 3) A description of minimal level of care a patient is expected to receive 4) Not part of the care plan that is included in the patient's chart

1) Written for a specific medical diagnosis or treatment 3) A description of minimal level of care a patient is expected to receive Rationale: Unit standards describe the minimal level of care the nurses are expected to achieve. They are not kept in the patient's chart but are kept in a file on the unit. Protocols are written to cover specific medical diagnoses and treatments. Unit standards are not organized according to nursing diagnoses.

A patient is obese wants to lose weight and start exercising. She complaints of fatigue with activity. Which of the following nursing interventions would be appropriate for this patient? Encourage her to (select all that apply): 1) consult her primary care provider prior to starting a fitness program 2) increase her level of activity within her daily routine. 3) refuel with protein and carbohydrate within 45 minute of her workout 4) rest as needed when her perceived level of exertion is high

1) consult her primary care provider prior to starting a fitness program 2) increase her level of activity within her daily routine 3) refuel within 45 minute of finishing her workout. 4) rest as needed when she is too fatigued to talk while exercising Rationale: This patient can begin to improve her activity tolerance by increasing her level of physical activity with everyday activities, such as taking stairs instead of an elevator, or parking farther rather than in the closest spot. A regular exercise program has many benefits, including weight loss and improving overall cardiovascular health. However, to reduce the risk of injury and other health complications, she needs to be seen by primary care provide for a thorough history and physical assessment. If she has not been exercising regularly, it is best to start out slowly and gradually increase the intensity and frequency of her workout, and rest as needed when her perceived level of exertion is high. Use the Talk Test to discover when she is too tired to talk while exercising. After a moderate or vigorous intensity workout, the body needs to be refueled with proteins and carbohydrates, regardless of the person's baseline weight (i.e., whether the person is a normal weight or obese). Good hydration, especially water, not only quenches thirst but also replaces the fluid lost by exertion and perspiration.

What are the four parts of a NANDA-I nursing diagnosis?

1. Label 2. Definition 3. Defining characteristics 4. Related or risk factors

To promote exercise for a patient who is obese, the nurse's best statement would be: 1) "I know exercising is hard, but you are going to have to do it to remain healthy." 2) "Gaining weight and getting out of shape take time; reversing that also takes time." 3) "I know exactly how you feel; I absolutely hate to exercise, too." 4) "The problem was probably the exercise program you participated in."

2) "Gaining weight and getting out of shape take time; reversing that also takes time." Rationale: Many patients feel they have tried to exercise but that it did not make a difference, and they give up. Remind them that gaining weight and getting out of shape take time and that reversing these changes also takes time. Patients may not begin to see the effects of an exercise program for at least 12 weeks. Encourage them to be consistent and not give up.

What is wrong with the format of this diagnostic statement: Possible Risk for Constipation related to irregular defecation habits A.M.B. verbalizing that "When I'm busy, I can't always take the time to go to the bathroom." 1) Possible nursing diagnoses do not have signs and symptoms. 2) A nursing diagnosis is either possible or it is a risk, not both. 3) Risk for Constipation is a medical diagnosis. 4) The etiology is actually a defining characteristic.

2) A nursing diagnosis is either possible or it is a risk, but not both Rationale: If there are risk factors, it is not a possible diagnosis; it is a risk diagnosis. It is not possible to have a "possible risk for" diagnosis. The patient with possible diagnoses may have symptoms, just not enough to support the diagnosis. Constipation is a nursing diagnosis, and the etiology is a defining characteristic for a risk diagnosis because it contributes to the problem. In risk diagnoses, the etiology consists of the risk factors.

The client has a nursing diagnosis of Impaired Physical Mobility. Which of the following is a Nursing Outcomes Classification (NOC) outcome label to use with this diagnosis? 1) Increases his physical activity 2) Activities of daily living 3) Demonstrates appropriate use of adaptive equipment 4) Verbalizes feeling of increased strength

2) Activities of daily living Rationale: NOC outcomes are broad neutral labels, not goal statements; the physical activity and strength outcomes are specific goals because they use the word increase, implying a target behavior. The adaptive equipment outcome includes the word appropriate, implying a judgment of the quality of the behavior—not a neutral statement. Goals state what the desired status will be. NOC outcomes provide only the topic to be evaluated.

The nurse has written this diagnosis for a patient: Ineffective Airway Clearance related to weak cough secondary to incisional pain. Which of the following outcomes is essential for the nurse to include in the care plan? 1) Effective cough 2) Airways clear to auscultation 3) Pain less than 4 on a scale of 1 to 10 4) Demonstrates splinting of incision

2) Airways clear to auscultation Rationale: For every nursing diagnosis, it is essential to have one goal that, if achieved, would show resolution of the problem. If the airways were clear to auscultation, that would show resolution of Ineffective Airway Clearance. Effective cough, reduced pain, and splinting of incision would help to resolve the problem, but they are aimed at the etiology side of the diagnostic statement. Even if pain is reduced, for example, the airways still might not be clear.

In what structure of the pulmonary system does inhaled air come in contact with the blood of the pulmonary circulation? 1) Apex of the lungs 2) Alveolar-capillary membrane 3) Cilia in the bronchi 4) Right and left main-stem bronchi

2) Alveolar-capillary membrane Rationale: The lungs are composed of millions of alveoli. The alveoli are tiny air sacs with thin walls surrounded by a fine network of capillaries. Gases easily pass back and forth between the alveoli and capillaries. It is at the alveolar-capillary membrane that inhaled air comes in contact with the blood of the pulmonary circulation.

For which patient would it be most important to perform a comprehensive discharge plan? 1) A teen who is a first-time mother, single, and lives with her parents 2) An older adult who has had a stroke affecting the left side of his body and lives alone 3) A middle-aged man who has had outpatient surgery on his knee and requires crutches 4) A young woman who was admitted to the hospital for observation following an accident

2) An older adult who has had a stroke affecting the left side of his body and lives alone Rationale: A comprehensive discharge plan should be developed for older adults and anyone who has complex needs, including self-care deficits, especially when they live alone and have no support within the home. The other patients do not have the complex needs of the older adult patient who has had a stroke that affects body function.

Nursing interventions to reduce the risk of clot formation in the legs include which of the following activities? Select all that apply. 1) Keep the patient's hips and knees flexed while the patient is in bed. 2) Apply compression devices (e.g., sequential compression devices). 3) Turn the patient frequently or encourage frequent position changes. 4) Promote adequate hydration by encouraging oral intake.

2) Apply compression devices (e.g., sequential compression devices). 3) Turn the patient frequently or encourage frequent position changes. 4) Promote adequate hydration by encouraging oral intake. Rationale: Antiembolism stockings and SCDs are frequently used in perioperative patients to promote venous return and prevent clot formation. Turn patients frequently; teach patients to change positions frequently. This prevents vessel injury from prolonged pressure in one position. Promote adequate hydration to keep the blood from becoming viscous ("thick"). Viscous blood clots more readily.

The nurse is caring for a woman who complains of an unpleasant, crawling and tingling sensation in her legs. She tells the nurse that she has an irresistible urge to move around that prevents her from falling asleep What does the nurse suggest that she try before resorting to medication for relief. Check all that apply. 1) Drink a caffeinated beverage. 2) Apply vibration to legs. 3) Massage the lower legs. 4) Try active release therapy. 5) Walk around and stretch legs. 6) Apply night splints to both legs.

2) Apply vibration to legs. 3) Massage the lower legs. 5) Walk around and stretch legs. Rationale: People with restless legs syndrome (RLS) should avoid stimulants (e.g., caffeine). Other self-care measures include walking, massaging, stretching, heat or cold compresses, medication, vibration, and acupressure. Active release therapy targets soft tissue to break up scar tissue and adhesions that are caused by overuse, or repetitive or pressure injuries. A night splint holds the foot with the toes pointed up and with the foot and ankle at a 90-degree angle. This position applies a constant, gentle stretch to the plantar fascia when it is inflamed (i.e., plantar fasciitis).

The nurse phones the laboratory to schedule a blood test for a hospitalized patient. She also schedules an x-ray for the same day. Both of these require the patient to be NPO (to have no food or liquids by mouth) for several hours prior to the tests. The nurse makes sure the patient has something to drink at the last possible moment before the patient is NPO. After the tests, when analyzing the urine output, she takes into consideration that the patient has had nothing to drink for 7 hours. This is a good example of: 1) Collaborating 2) Coordinating 3) Delegating care 4) Individualizing care

2) Coordinating Rationale: Scheduling treatments and activities with other departments and putting together bits and pieces of information (e.g., NPO and data about urine output) are examples of coordination of care. Collaborating means working with other professionals to plan, make decisions, or perform interventions. Delegating is the transfer of the responsibility for a task to another person while retaining accountability for the outcomes. Individualizing care means to make the nursing interventions specific for a particular patient's unique needs.

A 65-year-old woman presents to the emergency department with complaints of shortness of breath, diaphoresis, dizziness, jaw pain, and mild chest pain. She reports nausea, fatigue, and feeling anxious. Based on this assessment, an appropriate nursing diagnosis would be: 1) Ineffective Airway Clearance 2) Decreased Cardiac Output 3) Impaired Gas Exchange 4) Chronic Hypocarbia

2) Decreased Cardiac Output Rationale: Women who experience the physical consequences of Decreased Cardiac Output occurring from myocardial ischemia or infarction often have milder chest pain than do men, or sometimes none at all. They are thought to be more likely than men are to experience other symptoms, such as jaw or back pain, nausea, fatigue, and shortness of breath. Cardiac pain typically does not change with inhalation or exhalation. Hypocarbia (hypocapnia) is a low level of dissolved CO2 in the blood because of hyperventilation. In most cases, blood oxygen levels remain normal. Severe hypocarbia stimulates the nervous system, leading to muscle twitching or spasm (especially in the hands and feet) and numbness and tingling in the face and lips. There is no evidence of difficulty maintaining a clear airway; therefore, Ineffective Airway Clearance and Impaired Gas Exchange are not good choices.

In which step of the nursing process does the nurse analyze data and identify client problems? 1) Assessment 2) Diagnosis 3) Planning outcomes 4) Evaluation

2) Diagnosis Rationale: In the assessment phase, the nurse gathers data from many sources for analysis in the diagnosis phase. In the diagnosis phase, the nurse identifies the client's health status. In the planning outcomes phase, the nurse formulates goals and outcomes. In the evaluation phase, which occurs after implementing interventions, the nurse gathers data about the client's responses to nursing care to determine whether client outcomes were met.

Which nursing intervention is considered an independent intervention? 1) Administering 1 liter of dextrose 5% in normal saline solution at 100 mL/hour 2) Encouraging the postoperative client to perform coughing and deep-breathing exercises 3) Explaining his diet to the client; then communicating the teaching with the dietitian 4) Administering morphine sulfate 2 mg IV to the client with postoperative pain

2) Encouraging the postoperative client to perform coughing and deep-breathing exercises Rationale: Encouraging the postoperative client to perform coughing and deep-breathing exercises is an independent nursing intervention. An independent intervention is one that nurses are licensed to prescribe, perform, or delegate based on their skills and knowledge. Administering IV fluid or morphine sulfate is a dependent intervention; it requires an order from a physician or advanced practice nurse but is carried out by the nurse. Explaining to the client how sodium intake affects his heart failure and then communicating the teaching with the dietitian is an interdependent intervention, one that is carried out in collaboration with other healthcare team members.

The nurse has written the following diagnosis: Diarrhea r/t frequent loose stools. Which of the following describes the error in that diagnostic statement? 1) Diagnostic statement does not include an etiology. 2) Etiology does not describe the cause of the problem. 3) Statement includes a medical diagnosis. 4) Problem is stated as a need rather than a response.

2) Etiology does not describe the cause of the problem. Rationale: The etiology, frequent loose stools, should consist of related factors—that is, factors that are causing or contributing to the problem (Diarrhea). In this diagnostic statement, the etiology consists of defining characteristics—the symptoms by which the nurse can infer the diagnosis of Diarrhea. The statement does include an etiology (although an incorrect one), so that is not the error. Neither the problem nor the etiology contains a medical diagnosis, so that is not the error. The problem (Diarrhea) is, indeed, a patient response rather than a need; so that is not the error.

The nurse is caring for a patient who is experiencing dyspnea. Which of the following positions would be most effective if incorporated into the patient's care? 1) Supine 2) Head of bed elevated 80° 3) Head of bed elevated 30° 4) Lying on left side

2) Head of bed elevated 80° Rationale: Position affects ventilation. An upright or elevated position pulls abdominal organs down, thus allowing maximum diaphragm excursion and lung expansion.

Obesity is associated with higher risk for which of the following conditions that affect the pulmonary and cardiovascular systems? Select all that apply. 1) Reduced alveolar-capillary gas exchange 2) Heart failure 3) Atherosclerosis 4) Hypertension

2) Heart failure 3) Atherosclerosis 4) Hypertension Rationale: Adequate circulation ensures that oxygenated blood reaches tissues and organs and that venous blood returns to the heart. Obesity causes multiple health problems, including atherosclerosis, hypertension and heart failure.

An 85-year-old patient with hypertension (elevated blood pressure) was admitted to the hospital with dehydration. He has pressure sores on his back and hip. Which of the following tasks could the registered nurse delegate to nursing assistive personnel? Select all that apply. 1) Evaluating the healing of the pressure sores 2) Helping the patient to turn and reposition in bed 3) Coordinating the patient's care with the dietitian 4) Taking the patient's blood pressure and reporting changes in skin condition

2) Helping the patient to turn and reposition in bed 4) Taking the patient's blood pressure and reporting changes in skin condition Rationale: The RN can delegate specific tasks such as turning and positioning to the nursing assistive personnel (NAP). The nurse could also ask the NAP to take the patient's blood pressure and to let her know whether she notices any changes in his skin. The RN is responsible and accountable for evaluating responses to nursing interventions and for coordinating patient care.

What are the physical effects shown in the research that can result from chronic and long-standing sleep deprivation? Select all that apply. 1) Nicotine dependence 2) Insulin resistance 3) Stroke 4) Headache 5) Pain sensitivity 6) Infection

2) Insulin resistance 3) Stroke 4) Headache 5) Pain sensitivity 6) Infection Rationale: Lack of sleep increases the risk heart disease and stroke. Sleep restriction is also linked to increased appetite through changes in the appetite control hormones. In addition, not enough sleep leads to reduced energy expenditure, all of which can lead to obesity and type 2 diabetes. Headaches are common with sleep deprivation. Poor sleep intensifies the body's sensitivity to pain. Sleep is a time for rest and restoration. It strengthens the immune system and helps the body to fight infection. Nicotine can lead to poor sleep quality. Nicotine dependence generally is not the result of not getting enough sleep.

A patient is admitted with shortness of breath. The nurse specifically wants to know when the problem began, how often it occurs, what makes it worse, and what the patient has done to relieve it. What should the nurse do to obtain this information? 1. Complete a comprehensive admission assessment. 2. Interview the patient about the history of her present illness. 3. Perform a review of body systems and functional abilities. 4. Ask the patient about her expectations for care.

2) Interview the patient about the history of her present illness Rationale: The history of present illness is an in-depth exploration of the patient's chief complaint, which in this case you can assume is shortness of breath. In that exploration, the nurse will ask when the problem began, how often it occurs, what makes it worse, what the patient has done to relieve it, how the person's health has changed from her usual status, and what effect the illness has had on her daily life. A comprehensive admission assessment would include the history of the present illness, but would also include a great deal of other information. This question asks about specific data, which are found in the history of the present illness. A review of body systems might also produce the desired data, but it too would provide much additional data. Information about the patient's expectations for care might provide data about the patient's knowledge about the illness and what she wants the nurses to do; but it would likely not provide the specific data the question asks for.

Which statement about the nursing process is correct? 1) It was developed from the ANA Standards of Care. 2) It is a problem-solving method to guide nursing activities. 3) It is a linear process with separate, distinct steps. 4) It involves care that only the nurse will give.

2) It is a problem-solving method to guide nursing activities. Rationale: The nursing process is a problem-solving process that guides nursing actions. The ANA organizes its Standards of Care around the nursing process, but the process was not developed from the standards. The nursing process is cyclical and involves care the nurses give or delegate to other members of the healthcare team.

Which of the following is appropriate to the registered nurse's role in nursing diagnosis? 1) Decide when to delegate diagnosing to the LPN/LVN. 2) Make clinical judgments about the patient data. 3) Validate all nursing diagnoses with the primary care provider. 4) Use only NANDA-I standardized language to state problems.

2) Make clinical judgments about the patient data. Rationale: Professional standards and state nurse practice acts identify nursing diagnosis as the unique obligation of the professional nurse. As a rule nursing diagnosis cannot be delegated. Nurses should not validate nursing diagnoses with medical providers because they are not educated to make nursing diagnoses. If the nurse does not find the NANDA-I diagnostic labels to be helpful in describing a patient's problem, it is acceptable to describe the problem in other terminology; some electronic health records use other terminologies to describe nursing diagnoses.

Why is implementation known as the action phase of the nursing process? Because it is the stage in which the nurse: 1) Reassesses to see whether goals have been met 2) Performs or delegates the planned interventions 3) Documents the care that has been given 4) Prioritizes interventions

2) Performs or delegates the planned interventions Rationale: Implementation is known as the action phase of the nursing process because the nurse performs or delegates the planned interventions in the client's nursing care plan. Reassessing is a part of the evaluation phase. Assessing what is done as nursing care is given might be considered a part of implementation. However, determining whether goals are met is a part of the evaluation phase. Documentation occurs in the implementation phase; however, it does not reflect the full range of actions the nurse takes in implementing, so it does not provide the best answer to the question. Prioritizing is done in the planning stage.

The nurse writes a nursing diagnosis of Risk for Deficient Fluid volume for a 45-year-old patient admitted with acute pancreatitis. What type of nursing diagnosis has the nurse written for this patient? 1) Actual 2) Potential 3) Possible 4) Wellness

2) Potential Rationale: "Risk for" indicates a potential problem. Risk for Deficient Fluid Volume is a potential nursing diagnosis. Potential nursing diagnoses are designed to identify a patient health problem that could progress to become an actual problem. Based on the nurse's knowledge of acute pancreatitis, she knows that pancreatitis places the client at risk for Deficient Fluid Volume. Actual nursing diagnoses identify problems that already exist. They help detect changes in the client's health status. Possible nursing diagnoses help obtain more data to confirm or eliminate a suspected nursing diagnosis. Wellness diagnoses help assess the client's wellness practices.

Which part of the ECG tracing represents ventricular repolarization? 1) P wave 2) QRS complex 3) T wave 4) U wave

2) QRS complex Rationale: The QRS complex represents ventricular depolarization and leads to ventricular contraction. The P wave represents the firing of the SA node and conduction of the impulse through the atria. In the healthy heart, this leads to atrial contraction. The T wave represents the return of the ventricles to an electrical resting state, so they can be stimulated again (ventricular repolarization). The atria also repolarize, but they do so during the time of ventricular depolarization; thus, they are obscured by the QRS complex and cannot be seen on the ECG complex. The U wave is not always seen on the ECG, but may be detected with electrolyte imbalance, such as hypokalemia or hypercalcemia. U waves sometimes occur in response to certain medication (e.g., digitalis, epinephrine). Inverted U wave may occur with ischemia to the cardiac muscle.

The nurse is preparing to admit a patient from the emergency department. The patient has chronic lung disease and has used tobacco for 30+ years. The nurse used to smoke a pack of cigarettes a day and worked very hard to quit smoking. She thinks to herself, "I know I tend to disapprove of people who use tobacco, especially when they have a serious lung condition; I figure if I can stop smoking, they should be able to. I must remember how difficult that is, and be very careful not to let be judgmental of this patient." This best illustrates: 1) Theoretical knowledge 2) Self-knowledge 3) Using reliable resources 4) Use of the nursing process

2) Self-knowledge Rationale: Personal knowledge is self-understanding—awareness of one's beliefs, values, biases, and so on. That best describes the nurse's awareness that her bias can affect her patient care. Theoretical knowledge consists of information, facts, principles, and theories in nursing and related disciplines; it consists of research findings and rationally constructed explanations of phenomena. Using reliable resources is a critical-thinking skill. The nursing process is a problem-solving process consisting of the steps of assessing, diagnosing, planning outcomes, planning interventions, implementing, and evaluating. The nurse has not yet met this patient, so she could not have begun the nursing process.

Which is the best example of a well-stated desired outcome? The patient will: 1) Use the incentive spirometer while awake 2) State pain < 4 on a scale of 1 to 10 within 1 hour after receiving pain medication 3) Increase the distance he walks each time he ambulates 4) Verbalize the side effects of his new medication

2) State pain < 4 on a scale of 1 to 10 within 1 hour after receiving pain medication Rationale: Outcome criteria should be stated with specific measurement criteria; the pain outcome is the most specific and measurable or observable. "Use incentive spirometer" should specify how long and how often. "Increase the distance" should say how much he should increase the difference and what the baseline distance is. "Verbalize side effects" should specify by what date the patient is expected to do this.

Which of the following is an example of data that should be validated? 1) The urinalysis report indicates there are white blood cells in the urine. 2) The client states she feels feverish; you measure the oral temperature at 98°F. 3) The client has clear breath sounds; you count a respiratory rate of 18 breaths/min. 4) The chest x-ray report indicates the client has pneumonia in the right lower lobe.

2) The client states she feels feverish; you measure the oral temperature at 98°F. Rationale: Validation should be done when subjective and objective data do not make sense. For instance, data are inconsistent when the patient feels feverish and you obtain a normal temperature. The other distractors do not offer conflicting data. Validation is not usually necessary for laboratory test results.

What is the error in the following nursing order: "7-21-15—Using 2 persons, assist the patient from bed to chair 3 times per day. Jerry Xeno, RN"? 1) There is no action verb. 2) There are no times or limits. 3) Nurses do not need to sign nursing orders. 4) The order is too long and complex.

2) There are no times or limits. Rationale: The action verb is assist. Although the order includes instructions to use 2 persons to assist the patient out of bed, and to do that 3 times per day, it needs further clarification with regard to times and limits. That is, it does not say when to get the patient out of bed (e.g., every 8 hours, once per shift), and it does not say how long the patient is to remain in the chair. Nurses do need to sign nursing orders for accountability. The order is not too long and complex; in fact, it needs more information.

Which of the following are cues? Select all that apply. 1) Taking a brisk walk five times a week 2) Using laxatives to have a bowel movement 3) Needing more sleep than usual 4) Decreasing the amount of fat in the diet 5) Weighing less than indicated by developmental norms

2) Using laxatives... 3) Needing more sleep... 5) Weighing less than indicated by developmental norms Rationale: Cues are a deviation from norms, such as changes in usual health behavior, indications of delayed growth and development, changes in behaviors, or nonproductive or dysfunctional behavior. Taking a brisk walk five times a week and decreasing the fat in the diet are health promoting behaviors; they are normal, productive behaviors.

Using the mnemonic H E L P (from Volume 1 of your textbook), which of the following correctly illustrates the letter P—People? 1) Is the oxygen running? 2) Who else is in the room with the patient? 3) Take a thorough look at the patient. 4) Is the patient in pain?

2) Who else is in the room with the patient? Rationale: H: (Help): Look for signs of patient distress E: (Equipment and Environment): Check for safety hazards and equipment operation. L: (Look more closely): Look more closely at the patient for cues that care may need to be given. P: (People): Who else is in the room?

A patient who is prescribed a beta-adrenergic blocking agent asks the nurse why she needs to take this type of medication for her angina. The nurse tells her that this class of drug acts to: 1) increase pulmonary blood flow. 2) decrease myocardial oxygen demand. 3) improve conduction through the AV node. 4) remove sodium and water from the body.

2) decrease myocardial oxygen demand. Rationale: Beta-1 selective agents are used to treat angina, acute myocardial infarction, and congestive heart failure (CHF). They decrease heart rate, slow conduction through the AV node, and decrease myocardial oxygen demand by reducing myocardial contractility. Diuretics increase removal of sodium and water from the body by increasing urine output. Increasing pulmonary blood flow can lead to congestive heart failure.

The nurse in the hospital who is trained in cardiopulmonary resusitaiton (CPR) focuses on: 1) administering 15 compressions to 2 breaths for all victims except infants. 2) pushing hard in the center of the chest about 100 compressions/minute. 3) performing abdominal thrusts with finger sweep to remove obstruction in airway. 4) giving one long and forceful breath over 3 seconds for every 15 compressions.

2) pushing hard in the center of the chest about 100 compressions/minute. Rationale: Key points of the most recent guidelines for trained professionals include the following: Focus on effective, uninterrupted chest compressions. Push hard, push fast in the center of the chest. Administer about 100 compressions per minute. Perform 30 compressions to two breaths—for all victims except newborns. Give breath over 1 second and make the chest rise visibly (Dumas, Rea, Fahrenbruch, et al., 2013; Sayre, Berg, Cave, et al., 2008).

A middle-aged man has no known medical problems, although he is overweight. He tells the nurse that he is a computer programmer and gets little exercise. The nurse suggests to the patient that a fitness program would be beneficial to control his weight and improve his overall health. The patient states, "I would like to exercise but I am afraid I will have a heart attack and die if I strain my heart too much." The nurse's best response would be: 1) "It sounds like you are fearful of having a heart attack and dying. I understand why you might feel this way." 2) "Your records show there is a health history of heart attack in your family; tell me more about that." 3) "The risk of having a heart attack during strenuous exercise is low, but you still should be checked out first." 4) "The risk of having a heart attack during strenuous exercise increases in a high-risk patient such as you."

3) "The risk of having a heart attack during strenuous exercise is low, but you still should be checked out first." Rationale: Exercise itself is rarely life threatening, especially when compared with the alternative (not exercising). Before starting an exercise program or significantly increasing the intensity of normal workouts, conditioned athletes as well as novices should be screened for underlying health problems, such as high blood pressure, thickened heart muscle (cardiac hypertrophy), electrical abnormalities (dysrhythmias), and blood vessel abnormalities. Although showing empathy might help establish rapport and asking questions about family medical history might elicit information, neither does anything to address the patient's concern and encourage him to exercise. He is not a "high-risk" patient according to the data in this scenario and would not have a greater risk of heart attack with strenuous physical activity.

Arrange the steps of the nursing process in the sequence in which they generally occur. A. Assessment B. Evaluation C. Planning outcomes D. Planning interventions E. Diagnosis 1) E, B, A, D, C 2) A, B, C, D, E 3) A, E, C, D, B 4) D, A, B, E, C

3) A, E, C, D, B Rationale: Logically, the steps are assessment, diagnosis, planning outcomes, planning interventions, and evaluation. Keep in mind that steps are not always performed in this order, depending on the patient's needs, and that steps overlap.

A patient is obese and complains of fatigue with activity. She states, "I am so tired, I can't even walk out to my mailbox without resting every 5 minutes." Her vital signs are within normal limits at rest; however, upon minimal exertion she experiences dyspnea, and her pulse rises from 80 to 140 beats/min. She denies chest pain and has full range of motion to all joints. The nurse practitioner explains to the patient that she needs to lose weight and begin a fitness program. The best nursing diagnosis for this patient is: 1) Impaired Physical Mobility related to obesity 2) Risk for Disuse Syndrome related to musculoskeletal inactivity 3) Activity Intolerance related to obesity and dyspnea secondary to sedentary lifestyle 4) Impaired Physical Mobility related to limited range of motion, secondary to obesity

3) Activity Intolerance related to obesity and dyspnea secondary to sedentary lifestyle Rationale: In this scenario, Impaired Physical Mobility is the etiology of another problem: Activity Intolerance. Activity Intolerance is a state in which a patient has insufficient physical or psychological energy to carry out daily activities. Subjective characteristics include fatigue, weakness, discomfort with exertion, dyspnea, and verbalization of no interest in activity. Objective characteristics include changes in heart rate, blood pressure disproportionate to activity, dysrhythmias, or evidence of ischemia on electrocardiogram (ECG), and pallor or cyanosis with activity. There is no evidence that this patient has limited range of motion. Disuse syndrome is a more severe complication of immobility, more likely to be seen where there is little to no skeletal muscle activity.

You are to connect a patient to a cardiac monitor. Which of the following actions should you take to ensure an accurate electrocardiogram tracing? 1) Select electrode placement sites over bony prominences. 2) Apply the electrodes immediately after cleansing the skin, before the alcohol evaporates. 3) Before applying the electrodes, rub the placement sites with gauze until the skin reddens. 4) Ensure that the gel on the back of the electrodes is dry.

3) Before applying the electrodes, rub the placement sites with gauze until the skin reddens. Rationale: Rubbing the skin with gauze or a washcloth removes dead skin cells and promotes better electrical contact. Electrodes should be placed over soft tissues or close to bone in order to obtain accurate waveforms. Sites over bony prominences, thick muscles, and skinfolds can produce artifact; therefore, they should not be used. Alcohol removes skin oils that may prevent the electrodes from adhering. However, the alcohol should be allowed to dry before the electrodes are placed. A dry electrode will not conduct electrical activity; gel should not be dry.

Which of the following provides the most reliable data about the effectiveness of airway suctioning? 1) The amount, color, consistency and odor of secretions 2) The patient's tolerance for the procedure 3) Breath sounds, vital signs, and pulse oximetry noted before and after suctioning 4) The number of suctioning passes required to clear secretions

3) Breath sounds, vital signs, and pulse oximetry noted before and after suctioning Rationale: Breath sounds, vital signs, and oxygen saturation levels noted before and after suctioning provide data about the effectiveness of suctioning. Information about the amount and appearance of secretions provides data about the likelihood of airway infection and/or inflammation. Data about the patient's tolerance of suctioning provide information about the patient's overall condition. The number of suctioning passes required to clear the secretions provides information about the amount of secretions present.

Which of the following explains why it is important to have the correct etiology for a nursing diagnosis? The etiology: 1) Is the cause of the problem. 2) Cannot always be observed. 3) Directs nursing interventions. 4) Is an inference.

3) Directs nursing interventions Rationale: The etiology directs nursing interventions. If the incorrect etiology is given, the nursing care would not be appropriate for the client. The other statements are true but not a reason that it is important for the etiology to be correct.

The nurse has just finished documenting that he removed a patient's nasogastric tube. Which is the next logical step in the nursing process? 1) Assessment 2) Planning 3) Evaluation 4) Diagnosis

3) Evaluation Rationale: The implementation phase ends when you document nursing actions on the client's chart. Implementation evolves into the evaluation step when you document the client's response to your interventions. As a general rule, the steps in order are as follows: assessment diagnosis, planning outcomes, planning interventions, implementation, and evaluation.

The nurse is beginning discharge planning for an older adult with left-side weakness. All of the following are important, but which action is most important in ensuring that the discharge plan is successful? 1) Start planning at admission. 2) Involve the family members. 3) Get patient input when making the plan. 4) Involve the multidisciplinary team.

3) Get patient input when making the plan. Rationale: It is most important to get patient input. The discharge plan may be developed in a timely manner and involve the family and a multidisciplinary team, but if the patient does not agree with the plan, it will not be successful.

In diagnostic reasoning, the nurse does all of the following when analyzing and interpreting data. Which task occurs first? 1) Cluster cues. 2) Identify data gaps and inconsistencies. 3) Identify significant data. 4) Make inferences.

3) Identify significant data. Rationale: The steps occur generally in this order, keeping in mind that they overlap and that the nurse may move back and forth between them: Identify significant data, cluster cues, identify data gaps and inconsistencies, make inferences, and identify problem etiologies. It would be counterproductive to cluster cues before identifying significant data because the cue clusters would be unmanageable and might not reveal problems. (They would have large amounts of both normal and abnormal data in them.) It is easier to identify data gaps and inconsistencies after cues are clustered; however, it is sometimes evident when you are identifying significant cues. You should make inferences based on the cue clusters you create.

A 45-year-old woman arrives at the emergency department with complaints of shortness of breath, anxiety, dizziness, and numbness and tingling around her mouth. Her respirations are deep, at a rate of 28 breaths/min. Her lungs are clear with good aeration throughout. Oxygen saturation is 100%. An arterial blood gas shows a PO2 of 110 and PCO2 of 29 mm Hg. Based on this assessment, an appropriate nursing diagnosis would be: 1) Ineffective Airway Clearance 2) Decreased Cardiac Output 3) Impaired Gas Exchange 4) Hypocarbia

3) Impaired Gas Exchange Rationale: Hypocarbia (hypocapnia) is a low level of dissolved CO2 in the blood because of hyperventilation. In most cases, blood oxygen levels remain normal. Severe hypocarbia stimulates the nervous system, leading to muscle twitching or spasm (especially in the hands and feet) and numbness and tingling in the face and lips. This patient is experiencing hypocarbia; however, this is a medical diagnosis. A corresponding nursing diagnosis is Impaired Gas Exchange. A person with Impaired Gas Exchange may very well be anxious. However, there are no defining characteristics given for Anxiety in this scenario.

Which of the following aids the nurse in best meeting the unique needs of a patient? 1) Kardex 2) Critical pathway 3) Individualized patient care plan 4) Standardized (model) patient care plan

3) Individualized patient care plan Rationale: Individualized, or nursing diagnosis, care plans address nursing diagnoses unique to a particular client. They reflect the independent component of nursing practice and include goals and nursing orders written specifically for a patient. A Kardex contains essential client data that either do not change or that are used and updated often. It includes ADLs and other basic care needs. It does not address a client's individual problems. A critical pathway is a standardized plan that specifies patient outcomes and broad interventions for all patients with a particular condition (e.g., myocardial infarction, sepsis). A standardized (model) patient care plan describes the nursing care that is usually needed for a particular nursing diagnosis or for all nursing diagnoses that commonly occur with a medical diagnosis. They do not describe care for a particular patient.

Chest percussion and postural drainage would be an appropriate intervention for which of the following conditions? 1) Congestive heart failure 2) Pulmonary edema 3) Pneumonia 4) Pulmonary embolus

3) Pneumonia Rationale: Chest physiotherapy moves secretions to the large, central airways for expectoration or suctioning. This treatment is not effective for conditions that do not involve the development of airway secretions, including congestive heart failure, pulmonary edema, and pulmonary embolus.

Which nurse is most clearly using evidence-based practice? One who uses an intervention: 1) He read about in a study in a nursing research journal 2) From the agency's critical pathway in the electronic health record 3) Published in the clinical practice guidelines of a national organization 4) That is individualized to meet a specific patient need

3) Published in the clinical practice guidelines of a national organization Rationale: Clinical practice guidelines provide the best evidence because they are usually developed by scientists and qualified healthcare professionals after a thorough review and evaluation of the available evidence for an intervention. An individual study published in a nursing journal can provide some applicable evidence; however, it might not be as scientifically sound as a body of studies of the same intervention. Critical pathways are not always evidence based. The nurse should always individualize interventions, as needed, to meet specific patient needs; however, this answer does not state that the intervention being individualized has any basis in evidence.

How is a critical pathway different from a standardized care plan? A critical pathway: 1) Does not include medical orders 2) Provides individualized goals and interventions 3) Specifies patient outcomes and interventions for each day, or other period of time 4) Is usually a preprinted document for a particular diagnosis or condition

3) Specifies patient outcomes and interventions for each day, or other period of time Rationale: A critical pathway specifies patient outcomes and interventions for each day, and in some situations (such as labor and delivery) for each hour. Critical pathways do include medical orders, so that statement is not true of critical pathways. Critical pathways are standardized, not individualized, so this is not true of critical pathways. Both critical pathways and standardized care plans are preprinted documents for a particular diagnosis or condition, so that statement cannot be used to differentiate them.

The nurse in the hospital has a prescription to administer medication at 0400 to Mrs. Giovanni. Mrs. Giovanni is asleep when the nurse enters the room. She is difficult to arouse and confused. Identify the stage of sleep Mrs. Giovanni was likely in when the nurse awakened her. 1) Stage I 2) Stage II 3) Stage III 4) REM

3) Stage III Rationale: Stage III is the deepest sleep and the most restorative. In this stage, the delta waves are highest in amplitude, slowest in frequency, and highly synchronized. The body, mind, and muscles are very relaxed. It is difficult to awaken someone in stage III sleep, and if awakened, the person may appear confused and react slowly.

The criterion reads: "Hallways clear and free of equipment (e.g., beds, wheelchairs)." This is an example of a criterion that would be appropriate in which type of evaluation? 1) Outcomes 2) Processes 3) Structures 4) Intermittent

3) Structures Rationale: Structures evaluation focuses on the setting in which care is provided. It explores the effect of organization characteristics on the quality of care. It requires standards and data about policies, procedures, fiscal resources, physical facilities and equipment, and number and qualifications of personnel. Outcomes evaluation focuses on demonstrable changes in the patient's health status that result from the care provided. Processes evaluation focuses on the manner in which care is given—the activities performed by nurses and other personnel. It explores whether the care was relevant to patient needs, appropriate, complete, and timely. Intermittent evaluation is performed at specified times and enables one to judge progress toward goal achievement; no times were specified in this question.

What is missing from this goal statement? "The patient will walk to the doorway with the help of one person." 1) Action verb 2) Special conditions 3) Target time 4) Nothing is wrong with it.

3) Target time Rationale: The goal does not have a target time. Its action verb is "will walk." Its special conditions are "with the help of one person."

How would the nurse be able to identify the person with narcolepsy from one with seizures? 1) Episodes are short in duration. 2) Episodes come on suddenly. 3) The patient can be aroused from the episode. 4) The patient loses voluntary control of his muscles.

3) The patient can be aroused from the episode. Rationale: The patient with narcolepsy can be aroused from the sleep episode. A person with seizure activity is unresponsive to stimulus and does not resolve in relationship to arousing. Narcolepsy and seizures are triggered suddenly. Both involve involuntary control of motor function with paralysis and cataplexy. Typical seizures last less than 8 minutes. Most narcoleptic episodes are also brief, with microactivity lasting only a few minutes. Infrequently, the uncontrollable urge to sleep goes on for up to an hour.

Depression, hyperthyroidism, hypothyroidism, pain, and sleep apnea are examples of: 1) disorders that are provoked by sleep. 2) conditions known as parasomnias. 3) conditions that cause secondary sleep disorders. 4) disorders associated with narcolepsy.

3) conditions that cause secondary sleep disorders. Rationale: Secondary sleep disorders occur when a disease causes alterations in sleep stages or in quantity of sleep. Depressed people may spend more time in bed; however, in general, they have difficulty falling asleep, experience less slow-wave (deep) sleep, spend less time in REM sleep, awaken early, and have less total sleep time. An increase in thyroid secretion causes an increase in stage III and IV sleep. Hypothyroidism causes a decrease in those stages. Hyperthyroidism creates increased metabolic rate, making it difficult to fall asleep. Acute pain and chronic pain interfere with sleep. They inhibit sleep, increase arousals during sleep, and cause longer awake intervals during the night. During periods of sleep apnea, O2 level in the blood drops, and the CO2 level rises, causing the person to wake up frequently.

While caring for a young adult being mechanically ventilated, the ventilator alarms sound. On entering the patient's room, the nurse notes that he is agitated and his skin is ashen and diaphoretic. His pulse oximetry shows an oxygen saturation of 78%. The nurse is unable to identify any obvious mechanical problems with the ventilator. The first step the nurse should take is to: 1) assess his breath sounds. 2) call the respiratory therapist to troubleshoot the problem. 3) manually ventilate him with an Ambu bag. 4) contact the physician.

3) manually ventilate him with an Ambu bag. Rationale: All the actions listed are appropriate and necessary. However, if you cannot quickly identify and correct a problem with the ventilator, you must ensure adequate ventilation until the problem can be identified and corrected. Your immediate response should be to manually ventilate the patient with an Ambu bag. Your colleagues should assist you by troubleshooting the problem, assessing breath sounds, and notifying the physician.

Which of the following is a collaborative intervention? 1) Rubbing patient's back to facilitate relaxation 2) Measuring the patient's blood pressure 3) Assessing the patient's educational needs related to discharge 4) Administering prescribed medications to a patient

4) Administering prescribed medications to a patient Rationale: Administering medications requires a physician's order and pharmacist to dispense; it is therefore a collaborative intervention for the nurse. The other options are independent nursing actions.

As the nurse is inserting a urinary catheter she observes blood in the urine returned in the tubing. Which principle of assessment does this best illustrate? 1. Assessment must be accurate because the remainder of the nursing process depends on it. 2. Nursing assessments focus on patient responses rather than disease processes. 3. Assessment is a responsibility of professional nurses and cannot be delegated. 4. Assessment is related to and overlaps with other steps of the nursing process.

4) Assessment is related to and overlaps with other steps of the nursing process. Rationale: The scenario best illustrates that assessment is related to and overlaps with other steps of the nursing process. In this case, while the nurse is implementing (inserting a catheter), she has the opportunity to assess the patient (observe the appearance of the urine). Therefore, implementation provides the opportunity for assessment (the two steps are related) and assessment occurs during (overlaps with) implementation. The scenario says nothing about other nursing process steps—only assessment and implementation. Although blood in the urine is a patient response, there is nothing in the scenario about disease processes. There is nothing in the scenario about delegation, nor whether this is a professional nurse inserting the catheter.

The nurse develops a goal on the nursing care plan that states: 9/26/16, 0900—By 0800 tomorrow, will reach tube-feeding goal of 80 mL/hour. How can the nurse best collect data to evaluate this goal? (Note: Residual is the amount of undigested feeding remaining in the stomach; blood glucose levels are affected by calories ingested and digested.) 1) Weigh the client daily. 2) Monitor urine output hourly. 3) Obtain blood glucose levels every 6 hours. 4) Check feeding residual every 2 hours.

4) Check feeding residual every 2 hours. Rationale: Checking tube-feeding residual every 2 hours helps assess the patient's ability to tolerate the tube feeding. This tells you whether the client can meet the tube-feeding goal of an administration rate of 80 mL/hour. Weighing the client, monitoring urine output, and obtaining blood glucose levels help monitor the client's nutritional status but do not specifically evaluate whether the tube-feeding administration goal is being achieved.

A clinic client has not been keeping his scheduled follow-up appointments. In talking with him about that, the nurse asks, "Do you have a car or other transportation to bring you to the clinic?" Which critical-thinking process does that question illustrate? 1) Inquiry based on credible sources 2) Reflective skepticism 3) Analyzing assumptions 4) Contextual awareness

4) Contextual awareness Rationale: Contextual awareness is noticing what is happening in the total situation, including environmental influences (transportation). Inquiry is applying standards of good reasoning to your thinking (e.g., What framework should I use to organize my information?). Reflecting skeptically involves questioning, analyzing, and reflecting on the rationale for your decisions (e.g., In priority order, what should I do now and why?). Analyzing assumptions requires recognizing that you are making an assumption, and then examining the beliefs that underlie your choices.

Which of the following is the most obvious example of defining characteristics of the diagnosis Deficient Fluid Volume? 1) Increased metabolic rate 2) Effects of medications 3) History of falls 4) Decreased urine output

4) Decreased urine output Rationale: Defining characteristics are signs and symptoms (cues) of a problem. Decreased urine output is the most obvious defining characteristic for Deficient Fluid Volume. Increased metabolic rate could contribute to Deficient Fluid Volume, but not directly. Some medications might cause (be the etiology of) Deficient Fluid Volume, but they would not be a defining characteristic. History of falls is seemingly unrelated to Deficient Fluid Volume, although the falls and the Deficient Fluid Volume might both be symptoms of a medical diagnosis.

A patient has not achieved a desired outcome by the target date. What should the nurse do next when reviewing the care plan? 1) Reassess to see whether other outcomes have been met. 2) Implement new nursing interventions. 3) Write a new nursing diagnosis. 4) Determine whether the outcome was realistic.

4) Determine whether the outcome was realistic Rationale: In reviewing the care plan, the nurse would have first evaluated the assessment process and the nursing diagnoses. She would already have done that before evaluating the outcomes. Therefore, she would not reassess nor change the nursing diagnosis. The nurse should not jump to the conclusion that different interventions are necessary until she first evaluates the goals. If the outcome was unrealistic, then the lack of goal achievement was not the result of ineffective interventions.

A patient has a diagnosis Impaired Walking related to knee pain secondary to arthritis. The nurse has written the following set of nursing orders for that diagnosis. Provide passive range of motion to the affected knee 3 times per day. Assist the patient to walk to the bathroom and at least 3 times per day. Which of the following is a valid criticism of this set of orders? The orders 1) Are too long and complex. 2) Do not address the etiology of the nursing diagnosis. 3) Do not address the problem side of the nursing diagnosis. 4) Should not include the words "secondary to arthritis."

4) Do not address the etiology (knee pain) of the nursing diagnosis. Rationale: Although nursing interventions sometimes flow from the problem side of a diagnosis, there must always be some interventions that address the etiology of the diagnosis. Otherwise, the nurse is merely treating the symptoms, rather than removing factors that contribute to the problem. In this instance, if the patient's pain is treated, the diagnosis of Impaired Walking may no longer exist. These orders are not too long and complex. Also, it is acceptable to include a medical diagnosis, such as arthritis, if it aids in understanding the patient's problem and if it is preceded by the words "secondary to."

Which intervention depends almost entirely on the client's adhering to the therapy? 1) Inserting an intravenous catheter 2) Turning a client every 2 hours 3) Shortening a surgical drain 4) Following a low-fat, low-calorie diet

4) Following a low-fat, low-calorie diet Rationale: Instituting and adhering to a low-fat, low-calorie diet is an intervention that depends almost entirely on the client's adhering to the therapy. Client cooperation is necessary for performing the other interventions, but the interventions do not depend on the client to the same extent.

Which of the following is a benefit of standardized care plans, as defined in your text? Standardized care plans: 1) Apply to every patient on a particular unit 2) Include both medical and nursing orders 3) Specify patient outcomes for each day 4) Help ensure that important interventions are not overlooked

4) Help ensure that important interventions are not overlooked Rationale: Standardized care plans help promote consistency of care and ensure important interventions are not forgotten. They are not likely to apply to every patient on a unit because they are usually single-problem plans or are used with a particular medical diagnosis. Unlike protocols, they do not include medical orders. Unlike critical pathways, they do not specify predicted patient outcomes for each day.

After assessing a patient, the nurse is analyzing and synthesizing the data she obtained. She thinks, "I know the patient must sit up to breathe; his respirations are shallow and fast; and he is pale. What does this mean? What are some possible explanations for these symptoms?" Which of the following critical-thinking attitudes is the nurse illustrating? 1) Fair-mindedness 2) Independent thinking 3) Intellectual courage 4) Intellectual curiosity

4) Intellectual curiosity Rationale: Intellectual curiosity involves inquiry—thinking, "What if . . . ? How does this work? Why is this happening? What is causing this?" The nurse is not just collecting data and reporting them to someone else, but is trying to determine the meaning of the data. Fair-mindedness is more about making impartial judgments and treating all viewpoints fairly. Independent thinking involves listening to others and not believing everything you are told—not always "going along with the crowd." Intellectual courage involves examining your values and beliefs and being willing to rethink previously held beliefs—not becoming resistant to change.

Using the mnemonic H E L P (from Volume 1 of your textbook), which of the following correctly illustrates the letter H—Help? 1) Is the oxygen running? 2) Who else is in the room with the patient? 3) Are there any spills? 4) Is the patient in pain?

4) Is the patient in pain? Rationale: H: (Help): Look for signs of patient distress E: (Equipment and Environment): Check for safety hazards and equipment operation. L: (Look more closely): Look more closely at the patient for cues that care may need to be given. P: (People): Who else is in the room?

What is wrong with this nursing order? "3/10/2015. Provide measures to relieve anxiety at every patient contact. J. King, RN" 1) Lacks a target time 2) Does not contain a verb 3) Should not be signed 4) Is vaguely worded

4) Is vaguely worded Rationale: The nursing order is vaguely worded and is not individualized. It should specify which measures to use to relieve the patient's anxiety. Goals, not nursing interventions, need target times. In this order, provide is not an action verb. Nursing orders should be signed, so that is not an error.

To prevent weight gain in an adult who has lost a significant amount of weight, how much physical activity should the nurse recommend? 1) 60 to 119 minutes/week 2) 120 to 150 minutes/week 3) 151 to 250 minutes/week 4) More than 250 minutes/week

4) More than 250 minutes/week Rationale: For prevention of weight gain in most adults, adults are encouraged to engage in physical activity for 150 to 300 minutes per week with an energy equivalent of 1,200 to 2,000 kcal/week. However, when an adult has intentionally lost a significant amount of weight, more physical activity is needed to help minimize weight regain.

Which nursing intervention is best individualized to meet the needs of a specific client? 1) Suction the client every 2 hours per unit policy. 2) Use incentive spirometry every hour while awake per postoperative protocols. 3) Institute swallowing precautions. 4) Move client out of bed to the chair daily; client prefers to be out of bed for dinner.

4) Move client out of bed to the chair daily; client prefers to be out of bed for dinner. Rationale: Positioning the client in the chair for meals considers the client's desire to be out of bed for dinner, so it is obviously individualized. An intervention performed according to unit policy or protocols is not necessarily individualized. "Institute swallowing precautions" does not provide instructions for the specific actions needed to do that for "this particular" client.

What are the similarities between a risk nursing diagnosis and a possible nursing diagnosis? 1) Both are developed primarily from nursing intuition and experience. 2) Both require intervention from physicians. 3) Neither requires the client to have increased vulnerability. 4) Neither is made on the basis of client symptoms (defining characteristics).

4) Neither is made on the basis of client symptoms (defining characteristics) Rationale: A risk nursing diagnosis is used when a client is more susceptible (vulnerable) to the problem but does not have supporting data (e.g., signs and symptoms) that the problem exists, and risk factors are present. A possible nursing diagnosis is used when the nurse's intuition and experience make him or her suspect the diagnosis may be present, but again there are no (or minimal) supporting data and no increased vulnerability. Intuition is used for a possible nursing diagnosis, but not for a risk diagnosis. Collaborative problems, not nursing diagnoses, require physician intervention; so this option is not true for either a risk or a possible nursing diagnosis.

Which of the following goals is appropriate for a client without underlying cardiopulmonary disease who is being monitored with continuous pulse oximetry? 1) Patient will refrain from movement while monitored in order to ensure accurate readings. 2) Oxygen saturation will remain at 80% to 90% during hospitalization. 3) Patient will report pain as less than 3 on a scale of 1 to 10 during monitoring. 4) Oxygen saturation will remain at 95% to 100% while monitored.

4) Oxygen saturation will remain at 95% to 100% while monitored. Rationale: Normal oxygen saturation is 95% to 100%. Values of 94% or less are considered abnormal in healthy people and should be investigated to determine the cause. Although movement may affect oximetry monitoring, it is essential that patients move and turn in order to prevent atelectasis. Pulse oximetry is a noninvasive form of monitoring that involves placing a probe on a part of the body where capillary blood flow is near the surface (e.g., a nail bed, earlobe, nose, or forehead). It does not cause pain.

Which of the following goals is appropriate for a client with underlying heart disease who is being evaluated with continuous cardiorespiratory monitoring? 1) Patient will refrain from movement while monitored in order to ensure accurate readings. 2) Oxygen saturation will remain at 80% to 90% during hospitalization. 3) Patient will report pain as less than 3 on a scale of 1 to 10 during monitoring. 4) Patient will demonstrate normal sinus rhythm on ECG monitoring strip.

4) Patient will demonstrate normal sinus rhythm on ECG monitoring strip. Rationale: Normal oxygen saturation is 95% to 100%. Values of 94% or less are considered abnormal and should be investigated to determine the cause. Although movement may affect cardiorespiratory monitoring, it is essential that patients move and turn in order to prevent complications related to immobility (e.g., atelectasis, pressure injury to skin, fatigue, and muscle weakness). Monitoring with pulse oximetry is noninvasive that involves placing a probe on a part of the body where capillary blood flow is near the surface (e.g., a nailbed, earlobe, nose, or forehead). Cardiac monitoring is performed using electrodes (patches) that adhere to the skin. These do not cause pain.

What do standardized nursing care plans and individualized care plans have in common? They both: 1) Reflect critical thinking for a specific patient 2) Apply to needs common to a group of patients 3) Address a patient's individual needs 4) Provide detailed nursing interventions

4) Provide detailed nursing interventions Rationale: They both provide detailed nursing interventions, although the individualized care plan is more specific to the patient's needs and reflects critical thinking, whereas standardized plans do not. It is not true of individual nursing care plans that they are preprinted and apply to a group.

What is the role of The Joint Commission in regard to patient assessment? The Joint Commission 1. States what assessments are collected by individuals with different credentials 2. Regulates the time frames for when assessments should be completed 3. Identifies how data are to be collected and documented 4. Sets standards for what and when to assess the patient

4) Sets standards for what and when to assess the patient. Rationale: The Joint Commission sets detailed standards regarding what and when to assess, but does not address credentials. Nurse practice acts specify what data are collected and by whom. Agency policy may set time frames for when assessments should be conducted and how they should be documented. Nursing knowledge identifies "how" data are to be collected.

Which of the following nursing activities is a direct-care intervention? 1) Consulting with the nurse practitioner about a patient's medication 2) Telephoning the physician when a pain medication is not relieving the patient's pain 3) Checking and stocking the unit's resuscitation cart daily 4) Sitting with a patient who is anxious about his upcoming surgery

4) Sitting with a patient who is anxious about his upcoming surgery Rationale: Direct-care interventions are performed through interaction with the patient (e.g., patient teaching). Indirect-care interventions are performed away from the patient but on behalf of the patient or a group of patients.

Which of the following is an example of appropriate behavior when conducting a client interview? 1) Recording all the information on the agency-approved form during the interview 2) Asking the client, "Why did you think it was necessary to seek healthcare at this time?" 3) Using precise medical terminology when asking the client questions 4) Sitting, facing the client in a chair at the client's bedside, using active listening

4) Sitting, facing the client in a chair at the client's bedside, using active listening Rationale: Active listening should be used during an interview. The nurse should face the patient, have relaxed posture, and keep eye contact. Asking "why" may make the client defensive. Note taking interferes with eye contact. The client may not understand medical terminology or healthcare jargon.

Using the mnemonic H E L P (from Volume 1 of your textbook), which of the following correctly illustrates the letter L—Look? 1) Is the oxygen running? 2) Who else is in the room with the patient? 3) Are there any spills? 4) Take a thorough look at the patient.

4) Take a thorough look at the patient? Rationale: H: (Help): Look for signs of patient distress E: (Equipment and Environment): Check for safety hazards and equipment operation. L: (Look more closely): Look more closely at the patient for cues that care may need to be given. P: (People): Who else is in the room?

Which organization's standards require that all patients be assessed specifically for pain? 1) American Nurses Association (ANA) 2) State nurse practice acts 3) National Council of State Boards of Nursing (NCSBN) 4) The Joint Commission

4) The Joint Commission Rationale: The Joint Commission has developed assessment standards, including that all clients be assessed for pain. The ANA has developed standards for clinical practice, including those for assessment, but not specifically for pain. State nurse practice acts regulate nursing practice in individual states. The NCSBN asserts that the scope of nursing includes a comprehensive assessment but does not specifically include pain.

You are admitting a 54-year-old patient with chronic obstructive pulmonary disease (COPD). The physician prescribes O2 at 24% FIO2. What is the most appropriate oxygen delivery method for this patient? 1) Nonrebreather mask 2) Nasal cannula 3) Partial rebreather mask 4) Venturi mask

4) Venturi mask Rationale: The Venturi mask is capable of delivering 24% to 50% FIO2. The cone-shaped adapter at the base of the mask allows a precise level of FIO2 to be delivered. This is very useful for patients with chronic lung disease. Rebreather masks are used when high concentrations of oxygen are required. A nasal cannula administers oxygen in liters per minute and does not allow administration of a precise level of FIO2.

What assessment findings might the nurse expect to see in a patient with diabetes experiencing poor peripheral circulation? 1) Hairy feet and toes 2) Peripheral pitting edema 3) Inability to keep legs from moving 4) Weak or absent pedal pulses

4) Weak or absent pedal pulses Rationale: Poor peripheral circulation that occurs with diabetes is characterized by weak or absent pulses; pale, ashen, or cyanotic skin; and cool skin temperature. Lack of hair and shiny skin on lower legs and feet usually accompany peripheral vascular disease, such as what occurs with diabetes. Skin ulcers often accompany severe vascular disease. Peripheral edema does occur with heart failure.

A 10-year-old boy fell playing ice hockey. He comes to the emergency department reporting pain in his right forearm. The nurse notes that the boy's arm is swelling and tender to touch. There is an area of bruising on the lateral aspect of his arm, which upon palpation is very firm. The nurse suspects a fracture; however, it cannot be confirmed until: 1) a venous and arterial Doppler is obtained 2) clinical exam of the arm indicates a fracture 3) the swelling is decreased by applying ice 4) an x-ray of the extremity is obtained

4) an x-ray of the extremity is obtained Rationale: The variation in density evident on an x-ray allows the clinician to visually diagnose a fracture. A venous or arterial Doppler is used to detect blood flow and would not be used to detect a fracture. Although the clinical exam of the arm indicates a fracture, an x-ray of the suspected bone is a cost-effective, reliable test to definitively diagnose fracture. Applying ice would aid in decreasing the swelling at the site. Signs and symptoms associated with a sprain, strain, or fracture are the same

A 50-year-old woman had a surgical repair of a fracture of her right tibia 2 days ago. She has been using crutches for ambulation and must remain non-weight-bearing on her right leg, but must learn to use the steps leading into her house. The nurse should instruct the woman to: 1) set the crutches aside and use the stair rail to bear weight only on the left leg when going up or down stairs 2) use the crutches, maintaining toe-touch weight-bearing on the right leg when going up or down the stairs 3) have someone help her up and down the stairs by allowing her to hold on to that person 4) lead with the left leg when going up steps, and lead with the right leg when going down steps

4) lead with the left leg when going up steps, and lead with the right leg when going down steps Rationale: To teach the patient how to go down stairs, instruct him to hold his injured leg in front and hop down each stair on his good leg, one step at a time. When going up stairs with no handrail, he should lead with his good leg by standing close to the first step with weight on the crutches, and lift the uninjured leg, landing it solidly on the step. Then bring the crutches up to that same step, and repeat. If there is a handrail, then patient holds the crutches in one hand and handrail with the other. He then brings the good leg up one step, while the injured leg bears no weight.

A patient is obese and complains of fatigue with activity. To evaluate your nursing care plan for this patient, at the end of 12 weeks an indication of its effectiveness would be that she: 1) has lost 15 pounds when measured at the same time as the initial weight 2) has full range of motion to all joints without experiencing pain 3) jogs five city blocks twice a week without tiring or stopping 4) participates in a fitness program at the community center 3 days a week

4) participates in a fitness program at the community center 3 days a week Rationale: By participating regularly in an exercise program, the patient demonstrats an increase in activity tolerance (recall that her goal was Activity Tolerance). This is also an indication that her sedentary lifestyle habits are changing.

The duration of sleep is regulated by the: 1) electrical impulses transmitted to the cerebellum. 2) person's innate biorhythms. 3) amount of sleep a person usually requires. 4) reticular activating system.

4) reticular activating system. Rationale: In the morning, with an increase in environmental light, the hypothalamus is signaled to induce gradual arousal from sleep. The reticular formation is then activated by the stimuli from the cerebral cortex. The reticular formation is responsible for maintaining wakefulness. Together, the reticular formation and cortical neurons are called the reticular activating system (RAS). The RAS regulates the duration of sleep.

Which of the following is/are true about nursing diagnoses? They describe: A. a problem or strength B. an injury or illness C. a human response to disease, injury, or stressors

A & C Rationale: A nursing diagnosis is a statement of client health status that nurses can identify, prevent, or treat independently. It is stated in terms of human responses (reactions) to disease, injury, or other stressors, and can be either a problem or strength. Human responses can be biological, emotional, interpersonal, social, or spiritual.

The nurse is performing a morning assessment. The patient is curled on her side with the bed flat, and the nurse notes breath sounds are absent at the lung bases. Which of the following would be an appropriate nursing action? A. Assist the patient to sitting positionand reassess the lungs. B. Start oxygen at 4 L per nasal cannula C. Call the physician and notify him of the findings immediately. D. Call for a STAT respiratory treatment.

A. Assist the patient to sitting position and reassess her lungs. Rationale: The patient's position may be altering her ability to adequately perform deep breathing. Sitting the patient up would enable the nurse to more accurately assess the breath sounds before taking any other action.

George Mint, RN, has been assigned a 66-year-old female patient. Her diagnosis is congestive heart failure. Her blood pressure during the previous shift was in the normal range. The nursing assistive personnel (NAP) reports a very high current reading. Which of the following actions demonstrates the use of critical-thinking skills? A. George checks her blood pressure himself, and he considers possible causes if it remains elevated. B. George has the NAP record the blood pressure, and he plans to discuss the case with the charge later in the shift. C. George calls the doctor immediately to report these findings. D. George instructs the NAP to check the blood pressure in an hour and to notify him immediately if it remains elevated

A. George takes her blood pressure himself, and he considers the possible causes if it remains elevated. Rationale: Critical thinkers are inquisitive about information and facts. They wonder, "Why?" The patient had a normal blood pressure, and it is now elevated. Critical thinkers validate facts; George wishes to validate the NAP's data to be certain they are correct before trying to figure out causes for the problem. He wonders, "Why is the blood pressure elevated? Is the reading accurate? Or is something wrong?" Only in response A does the nurse demonstrate these skills. Response D might have been a second choice. If the NAP took the blood pressure an hour later and it had returned to normal, there might be no cause for concern. If the blood pressure was extremely high, though, the nurse would not want to wait another hour to be certain nothing was wrong.

The NANDA-I taxonomy is organized according to which of the following? A. Human response patterns B. Basic human needs C. Medical diagnoses D. Functional health patterns

A. Human response patterns

What is the most important reason to prepare before implementing the nursing plan of care? A. It allows the nurse to work efficiently. B. It allows the nurse to work systematically. C. It enables the nurse to predict patient outcomes. D. It enables the nurse to evaluate collaborative problems.

A. It allows the nurse to work efficiently. Rationale: Preparation improves efficiency in the delivery of care.

Which of the following could put a nurse or healthcare worker at risk for sustaining a back injury? A. Lifting a box of IV supplies up and over the head to place on a shelf B. Placing the feet 2 feet apart before sliding a patient up in bed C. Squatting to measure chest tube drainage D. Raising the bed to waist level when starting an IV

A. Lifting a box of IV supplies up and over the head to place on a shelf Rationale: Lifting an object overhead can strain a person's back or interfere with balance, leading to back injury or a fall. All other options represent the use of proper body mechanics.

Priority

After performing the nursing assessment and developing actual, possible, and/or risk nursing diagnoses, the nurse will record the diagnoses in the order of their ________.

Symptom

An organic or functional condition indicating the presence of disease.

Mary is working in a small rural hospital and is caring for a comatose patient who is beginning to develop foot drop. If there were no footboard available to use for proper positioning of the feet, Mary could use which of the following? A. Overhead trapeze B. Basketball shoes C. Pillow D. Trochanter roll

B. Basketball sneakers Rationale: Basketball shoes are a type of high-top, athletic footwear that help to keep the foot in flexion. The other responses are incorrect. A trapeze would not be used by a comatose patient. A pillow is too soft to maintain proper position of the feet, allowing rotation of the legs. A trochanter roll prevents external rotation of the hips.

Roberta, a 24-year-old woman, is admitted for treatment of burns to her left hand. She has some areas of blistering, and most of her hand is very red. She is able to move her fingers but reports severe pain with movement. The burns occurred when Roberta checked the bathwater that she had prepared for her 2-year-old child. The nurse admitting Roberta notices that she has a sad expression, occasionally wipes at tears, and is very quiet. In the admitting database, the nurse notes that Roberta asked if her baby would be taken away from her. The nurse asks Roberta if she is worried about the accident. This attitude represents which of the following? A. Intellectual curiosity B. Intellectual empathy C. Intellectual courage D. Fair-mindedness

B. Intellectual empathy Rationale: Intellectual empathy is trying to understand the feelings and perceptions of others. Intellectual curiosity is trying to figure out how and why things work. Intellectual courage is considering and examining one's own values. Fair-mindedness is treating all viewpoints fairly and not letting personal bias interfere with care.

Vital signs, level of consciousness, and skin color that you observe are which type of data? A. Secondary data B. Objective data C. Subjective data D. Focused data

B. Objective data Rationale: Objective data are observations and information collected about the patient's condition (e.g., vital signs). The other responses are incorrect. Secondary data are data that are obtained from any source other than the patient. Subjective data are patient statements and information supplied on the intake form. Focused data are in-depth information about abnormal cues or identifies problems with a body part or function.

Which of the following standardized classification systems was specifically designed for community health nursing? A. Clinical Care Classification B. Omaha System C. Nursing Outcomes Classification D. Nursing Intervention Classification

B. Omaha System

A collaborative problem is a(n) ___________________ problem. A. Actual B. Potential C. Risk D. Medical

B. Potential Rationale: A collaborative problem is always a potential problem. If it becomes actual, then it is no longer a collaborative problem, but a medical diagnosis requiring physician intervention.

The RN is assigning the care of patients to her team members: another RN, an LPN, and two NAPs. Mr. Bonn, a new patient, is scheduled for an angiogram later that day. He is diabetic and speaks only Spanish. The only staff member who speaks Spanish is one of the NAPs. The RN assigns Mr. Bonn to the other RN on the team. The RN makes this decision based on which of the following? A. Personal knowledge B. Theoretical knowledge C. Fair-mindedness D. Ethical knowledge

B. Theoretical knowledge Rationale: The nurse has used theoretical knowledge to evaluate the complexity of the patient's needs and assigned his care to the staff member with the scope of practice to do the nursing assessment, preprocedure teaching, and delegate aspects of his care. Theoretical knowledge consists of facts, principles, and theories in nursing. Personal knowledge consists of awareness of one's own attitudes, biases, values, and so on. Fair-mindedness is treating all viewpoints fairly and not letting personal bias interfere with care decisions. Ethical knowledge is the knowledge of obligation and duty, of right/wrong, of good/bad (e.g., not lying to someone).

The goal for your 17-year-old patient, Bill, who is 5'5" tall, is "Maintains current weight of 135 lb." This is which of the following types of goals? A. Risk goal B. Wellness goal C. Essential goal D. Aggregate goal

B. Wellness goal Rationale: This is a wellness goal because the patient response is maintenance of the current healthy status of his weight.

Of the following, which is an example of an open-ended question? A. Do you live alone or with family? B. What problems have you had since your injury? C. Did you injure your hand at work? D. Where did the accident occur?

B. What problems have you had since your injury? Rationale: This question offers the patient the opportunity to express concerns or discuss changes in functioning that have occurred since the injury; it provides opportunity for further exploration. The other questions allow the patient to answer in brief or one-word responses, which limits the data obtained.

The structure(s) of the respiratory system most associated with the function of respiration is/are the: A. trachea. B. alveoli. C. bronchi. D. nose.

B. alveoli. Rationale: Respiration refers to the exchange of oxygen and carbon dioxide in the lungs. This occurs across the alveolar membrane.

Which brainstem center responds to pressure changes in blood vessels to regulate cardiac function and blood pressure? Baroreceptors Chemoreceptors

Baroreceptors Rationale: Baroreceptors located in the walls of the heart and blood vessels are sensitive to pressure changes.

The second step in the nursing process is nursing diagnosis. Nursing diagnosis is based on data that is which of the following? A. Arbitrary B. Inferential C. Accurate D. Problem oriented

C. Accurate Rationale: Data that are accurate and verifiable are essential to establishing the nursing diagnosis. Recall that nursing diagnoses may also be used to identify risks for problems or wellness concerns. As a result, "problem-oriented" is not an appropriate response.

Mrs. Williams has severe chronic obstructive pulmonary disease and becomes very short of breath when completing her ADLs. Which of the following nursing diagnoses would most accurately reflect Mrs. Williams's mobility problem? A. Ineffective Coping B. Impaired Physical Mobility C. Activity Intolerance D. Deficient Knowledge

C. Activity Intolerance Rationale: There is no data to suggest that the patient is not coping or has a knowledge deficit. Although she does have impaired mobility, the most accurate diagnosis is Activity Intolerance related to her poor oxygenation status.

You are assigned to Ms. Sims for the evening shift. She was admitted 6 hours ago with dehydration, nausea and vomiting, and a fever of 101°F (38.3°C). The NAP reports her vital signs as temperature 99°F; blood pressure 120/60 mm Hg; pulse 80 beats/min; respirations 20 breaths/min; and her intake as 500 mL and her output as 100 mL. She is currently sleeping. This is an example of which of the following components of the nursing process? A. Evaluation B. Implementation C. Assessment D. Planning: intervention

C. Assessment Rationale: Assessment is the portion of the nursing process during which data are collected. Evaluation is the final phase of the process, when you judge whether your actions have treated the patient's health problems. Implementation is the action phase, carrying out plans of care. Planning: intervention is developing a list of possible interventions based on your nursing knowledge.

Cathy Bess, an RN, is preparing her patient for surgery. The patient complains of a headache, and Cathy notes his last recorded blood pressure was 170/90 mm Hg. Yesterday, his blood pressure was 130/70 mm Hg. He is prescribed an antihypertensive medication. The unit was very demanding when Cathy reported for her shift. Two regular staff members were absent because of illness, so an LPN from a different unit was assigned to administer medications on Cathy's unit today. Cathy is concerned that the patient may not have received his medication this morning. Cathy is demonstrating which of the following? A. Intellectual curiosity B. Practical knowledge C. Critical thinking D. Full-spectrum nursing

C. Critical thinking Rationale: Cathy is using critical thinking, which includes the intellectual activities of problem-solving and decision making. Intellectual curiosity is being curious or wondering "how things work." Practical knowledge is knowing what to do, following policies and procedures. Full-spectrum nursing is using all of these components to provide quality nursing care.

Mr. Long has had surgical repair of an abdominal hernia. He will be discharged tomorrow. You are assigned as his nurse for the evening shift. To identify his discharge planning needs, you will perform a ______________ assessment. A. Special needs B. Psychosocial C. Focused D. Comprehensive

C. Focused Rationale: A focused assessment is performed to obtain data about an actual, potential, or possible problem that has been identified. It focuses on a particular topic, body part, or functional ability rather than on overall health status. A psychosocial assessment gathers information about lifestyle, normal coping patterns, understanding of the current illness, personality style, previous mental health disorders, recent stressors, major issues related to the illness, and mental status. A special needs assessment is a type of focused assessment. It provides in-depth information about a particular area of client functioning and often involves using a specially designed form (e.g., nutrition and pain). A comprehensive assessment (also called a global assessment, patient database, or nursing database) provides holistic information about the client's overall health status.

The nurse is caring for Mrs. Plank, who had abdominal surgery yesterday. Today, the nurse has Mrs. Plank splint her incision to enable a strong cough that will move respiratory secretions. This intervention is meeting the outcomes for which nursing diagnosis? A. Impaired Gas Exchange B. Ineffective Breathing Pattern C. Ineffective Airway Clearance D. Impaired Spontaneous Ventilation

C. Ineffective Airway Clearance Rationale: There is no evidence given that oxygen is not diffusing across the alveolar membrane, nor that there is an alteration in the patient's ventilation. The purpose of coughing is to clear the airway of secretions.

A patient is restrained at the wrists. At intervals, she is flexing her arm, pulling against the restraint tied to the bed. Technically, what type of exercise is this? A. Aerobic B. Isotonic C. Isometric D. Anaerobic

C. Isometric Rationale: Isometric exercises involve muscle contraction without motion. They are usually performed against an immovable surface or object—for example, pressing the hand against a wall, or pulling the wrist against a secured restraint.

Mrs. Chiang is admitted for treatment of severe emphysema and bronchitis. She has had difficulty sleeping each night. Nursing interventions to address this problem should be developed from which of the following? A. Patient teaching B. Priority needs C. Nursing diagnoses D. Collaborative problems

C. Nursing diagnoses Rationale: Although all the responses need to be assessed, nursing interventions flow directly from the nursing diagnosis. Priority needs help the nurse prioritize problems. Collaborative problems are those that are identified as potential problems. Patient teaching is planned based on the patient's educational needs, as identified during assessment/reassessment.

Taylor, a 7-year-old boy, is brought to the Urgent Care Center by his father. He is bleeding from a wound on the back of his head. You inspect the wound visually and assess that you will need to clean the wound area to determine the length, depth, and severity. What additional component of physical assessment will you perform while cleansing the wound? A. Auscultation B. Percussion C. Palpation D. Developmental

C. Palpation Rationale: With palpation, you will feel the skin around the wound to determine whether there is swelling or any foreign objects such as dirt, gravel, or twigs in or around the wound. You will also be able to evaluate pain, if elicited. The other responses are incorrect. Auscultation is listening for changes such as decreased breath sounds and heart rate. Percussion is using the fingers and hand to tap on areas of the body to assess for changes. Developmental assessment is important when working with children, but to gather data you will need to talk to the patient to assess verbal skills and comprehension.

Which of the following explains why high-flow oxygen therapy is contraindicated in patients with chronic obstructive pulmonary disease (COPD)? High levels of oxygen: A. take away the carbon dioxide drive to breathe. B. burn out the oxygen receptors in the lungs. C. prevent hypoxemia. D. prevent hypercarbia.

C. prevent hypoxemia. Rationale: Patients with COPD are no longer sensitive to high levels of carbon dioxide; therefore, their primary drive to breathe becomes hypoxia. High-flow oxygen therapy "fixes" hypoxemia, thereby taking away the secondary drive to breathe in the client with COPD.

Critical pathways

Care plans that focus on diagnosis-related groups (DRGs) and are organized on a time line to meet recommended lengths of stay are called ________.

List three skills or types of knowledge that nurses need for determining nursing interventions.

Clinical judgment Critical thinking Past experience Answers might also include theoretical, practical, personal, and ethical knowledge; psychomotor and nursing process skills.

List the three types of knowledge and skills used in the implementation phase of the nursing process.

Cognitive Interpersonal Psychomotor

Etiology

Contains the factors that cause, contribute to, or create a risk for the problem.

List the four components of full-spectrum nursing.

Critical thinking Nursing knowledge Nursing process Patient situation

You were assigned to the care of Sally Combs on the 2300 to 0700 shift yesterday. You had talked with her extensively about her home and any problems she anticipated upon discharge. You documented her concerns about her limited mobility and reach, but today you recall that you forgot to document her concerns about her ability to pay her bills. You should do which of the following? A. Document it in today's progress note, identifying data and time collected. B. Leave a note for the social worker explaining the patient's concerns. C. Add information to original database. D. A & B E. A & C

D. A & B Rationale: You obtained this information on a previous shift, and it should be documented validly and reliably. Because this information is important to the patient's discharge planning, it needs to be conveyed to the social worker to be incorporated into the discharge plan. As this is part of discharge assessment data, it is inappropriate to add it to the original database, and information cannot be added to previous documentation.

Which of the following activities is a nurse/physician collaborative intervention for Joan R.? (Assume that all the interventions are appropriate.) A. Rubbing the patient's back to facilitate relaxation B. Encouraging the patient to discuss her fears about surgery C. Assessing the patient's educational needs related to discharge D. Administering medication to reduce fever and offering Joan up to 500 mL of fluid per shift

D. Administering medication and offering fluids Rationale: Reducing the patient's fever helps to conserve body fluids, and increasing her fluid intake by mouth will supplement the IV fluids. Administering medications and IV fluids are physician orders that require nursing judgment when implementing.

To complete the process of evaluation, the nurse compares patient reassessment data with which of the following? A. Nursing care plan B. Assessment data C. Outcome criteria D. All of the above

D. All of the above Rationale: The process of evaluation involves thinking about the patient's current symptoms and human responses in relation to assessment data, the nursing plan of care, and outcomes criteria to determine whether goals have been met.

Joan R., a 65-year-old woman, was brought to the hospital by her husband. He reported that "she has barely eaten this week, been very quiet, and complains of dizziness, nausea, and a severe headache." Her medical diagnosis includes dehydration. The doctor orders IV fluids at 150 mL/hr. Nursing care includes assessing vital signs every 4 hours, monitoring fluid intake and output, and providing sips of water as tolerated. Which of the following describes the interventions to treat her fluid deficit? A. Physician prescribed B. Nursing prescribed C. Nurse/client prescribed D. Collaborative

D. Collaborative Rationale: Joan is experiencing a problem that requires collaborative care. There will be primary provider-prescribed interventions, such as IV fluids and medication. She will also benefit from nursing interventions, such as offering sips of water, monitoring body temperature, and administering medication as prescribed.

Medication assessment is part of the initial assessment. Patients are asked if they take vitamins or supplements in order to gather which of the following? A. Information about the patient's culture and ethnicity B. Data about the patient's health concerns, such as osteoporosis C. Information about the value the patient places on health D. Data that might reveal an interaction with prescribed medications

D. Data that might reveal an interaction with prescribed medications Rationale: Many vitamins and supplements interfere or interact with prescription medications to reduce effectiveness or contribute to side effects. Patients may take specific supplements as part of their cultural practices, to address health concerns, or because they want to maintain a certain level of health; but safety is the primary reason for gathering this information. Medication data would not necessarily provide any information about the patient's culture, osteoporosis, or the value the person places on health.

In addition to proper positioning, which of the following would be an important nursing measure for a patient who is immobile? A. Encouraging a low-calcium diet to prevent kidney stones B. Limiting fluid intake so she does not have to use the bedpan as frequently C. Encouraging the patient to lie still so he does not cause a blood clot to become dislodged D. Performing a skin assessment to dependent areas at least once every shift

D. Performing a skin assessment to dependent areas at least once every shift Rationale: An immobile patient is at risk for pressure-related injury to the skin, especially from the patient is incontinent or diaphoretic. Frailty (bony prominences) also increases the risk for pressure injury. Therefore, a skin assessment at least once a shift is important for the early detection of decubitus. Other responses are incorrect. A diet low in calcium will not prevent kidney stones; kidney stones develop only in susceptible people, regardless of calcium intake. Limiting the fluid intake will place the patient at risk for a urinary tract infection. Keeping extremities still will lead to increased venous pooling and risk for the development of blood clots.

Delegation of nursing care is regulated legally by which of the following? A. ANA B. Hospital policy C. The Joint Commission D. State practice acts

D. State practice acts Rationale: State practice acts legally define the scope and practice of nursing within the state, and this is what you are accountable for. Hospital policy, ANA, and The Joint Commission all provide standards, guidelines, or positions for nurses to refer to for clarification as to the nurse's role or responsibility in the nursing process.

When developing patient care plans, it is important to remember that nursing orders include which of the following? A. Outcomes B. Theories C. Collaborative problems D. Strategies

D. Strategies Rationale: Strategies are the activities for enacting the nursing interventions, whereas collaborative problems are developed in the diagnostic phase. Outcomes are the goals to be achieved with intervention. Theories are the frameworks we use as a knowledge base to develop our plans of care.

You are preparing to perform an initial nursing assessment for 89-year-old Lucy J. She is petite, thin, and is seated in a wheelchair in her assigned room. As you enter the room, you observe her sobbing into a tissue. Your initial intervention to help her prepare for the interview is to do which of the following? A. Ask her if she would like you to call a family member for her. B. Ask her if she would like to talk about what is upsetting her. C. Walk over to Lucy, take her hand, and reassure her that things will be okay. D. Walk over to Lucy and introduce yourself by name, position, and role.

D. Walk over to Lucy and introduce yourself by name, position, and role. Rationale: Before beginning any nursing care with a new patient, introduce yourself by name and position (RN), and tell the patient what aspects of care you will be responsible for. None of the other responses is the first thing to do. Asking about calling a family member assumes information about her support system that may not be accurate; she may live in another state or be estranged from her family. Asking if she'd like to talk about what is upsetting her can be supportive, but this does not take into consideration any cultural barriers that may inhibit her from being comfortable talking to you about her personal feelings. Offering her reassurance is not an appropriate response; you have not collected enough data to determine what she is upset about.

The American Nurses Association's Code of Ethics for Nursing—Provisions (2008) states, "The nurse determines the appropriate ______________ of tasks consistent with the nurse's obligation to provide optimum ___________."

Delegation, patient care

hyperventilating

Describe the breathing of someone who is anxious and breathing deeply and rapidly. The person is _________________________.

Problem

Describes the client's health status (or a human response to a health problem) and identifies a response that needs to be changed.

Jose, a 47-year-old Hispanic male, is brought to the emergency department by ambulance. He is reported to have fallen and hit his head at his construction work site today. He is accompanied by an emergency medical technician (EMT) and a coworker. The coworker has a copy of the accident report completed by the company's RN. To obtain credible information for the nursing assessment, the nurse should gather data from which of the following? A. EMT B. Coworker C. Patient D. Accident report E. All of the above

E. All of the above Rationale: The EMT is trained and qualified to complete assessments and gather medical data; the coworker may have witnessed the accident or be able to provide information about Jose's pre-accident health; asking the patient questions will provide data about his mental status; and the accident report was completed by an RN and can provide assessment data for comparison.

In which phase of the nursing process does the nurse decide whether her actions have successfully treated the client's health problem? ________________

Evaluation Rationale: In the assessment phase, the nurse gathers data from many sources for analysis in the diagnosis phase. In the diagnosis phase, the nurse identifies the client's health status. In the planning outcomes phase, the nurse and client decide on goals they want to achieve. In the planning interventions phase, the nurse identifies specific interventions to help achieve the identified goal. During the implementation phase, the nurse carries out the interventions or delegates them to other healthcare team members. During the evaluation phase, the nurse judges whether her actions have been successful in treating or preventing the identified client health problem.

Nursing research studies have little effect on the quality of care.

False

Outcome identification is the description of the activities needed for achieving both short-term and long-term goals.

False

Pulse oximetry is the most accurate way to assess blood oxygen levels.

False Rationale: Arterial blood gases directly measure blood oxygen levels. Although reliable, a pulse oximetry reading is an indirectly calculated estimate.

Nurses manage collaborative problems using only physician-prescribed interventions.

False Rationale: Collaborative problems are managed with combinations of interventions from a variety of services, and all nursing problems will be treated with nursing interventions in conjunction with other prescribed interventions.

Students should disregard their current learning skills in order to learn the skill of critical thinking.

False Rationale: Critical-thinking skills build on the foundation of a person's current learning and thinking skills.

The nurse has developed and implemented a care plan for a patient. Evaluation of the patient's responses ends the nursing process for this patient.

False Rationale: Evaluation does not end the nursing process. The nursing process is continuous; evaluation provides information to begin another cycle of care.

The process of nursing diagnosis may be delegated to the licensed practical nurse. True or False?

False Rationale: In 1980, the ANA published Nursing: A Social Policy Statement, which characterized nursing as "the diagnosis and treatment of human response to actual or potential health problems" (ANA, 1980). As a result of this definition and the work of the nursing diagnosis task force, most states' nurse practice acts began to designate nursing diagnosis as an exclusive responsibility of registered professional nurses.

Flexibility training involves moving a muscle against resistance

False Rationale: Moving a muscle against resistance is resistance training.

Nursing care is regulated by national standards and institutional policies.

False Rationale: State nurse practice acts regulate the practice of nursing; standards and policies are guidelines that serve to promote high-quality standardized care.

The nursing process is linear, flowing from one step to the next.

False Rationale: The nursing process is cyclical, following a logical progression. However, the steps overlap, and the nurse moves back and forth between them.

The reason for monitoring nursing interventions is to help a client retain his present state of health.

False Rationale: The reason for monitoring interventions is to evaluate the client's changes in health status and to revise or maintain the nursing plan of care as needed.

Realistic

Goals that are specific, concrete, and measurable; valued by the patient; and are stated in terms of patient response or behavior are __________.

Nursing Diagnosis is a statement of a client's health status that nurses can ___________, ___________, or___________ independently.

Identity, prevent, or treat

orthopneic position

Leaning over a pillow on a table

Mrs. Jones becomes short of breath when the head of her bed is lower than 15° of elevation. Mrs. Jones is experiencing _________________________.

Orthopnea

"Essential" goals are derived from the ________clause of the nursing diagnosis.

Problem

Evaluation can focus on one of three areas: structure, ________, or ________.

Process, outcomes

List the "five rights" of delegation.

Right person Right task Right circumstance Right direction/communication Right supervision

What part of the cardiac conduction system is considered the pacemaker of the heart? AV node SA node Bundle of His Purkinje fibers

SA node Rationale: The SA node acts as the pacemaker. Located in the right atrium, it initiates an impulse that triggers each heartbeat. The impulse travels rapidly down the atrial conduction system so that both atria contract as a unit.

A patient with air in the pleural space whose chest tube becomes kinked or clamped is at risk for a __________ _____________.

Tension Pneumothorax

Identify five factors that influence the prioritization of nursing problems.

The patient's condition New data from reassessment Time and resources available for the nursing interventions The nurse's level of expertise and experience in assessment and setting priorities Feedback from the patient, family, or other healthcare workers Evidence base

A nurse is in the process of generating and selecting nursing interventions for a patient. Number the following steps in the order in which they should occur. 1) Choose the best interventions for the patient. 2) Identify several interventions or actions. 3) Review the desired patient outcomes. 4) Review the nursing diagnosis. 5) Individualize standardized interventions to meet the patient's unique needs.

The steps should be done in the following order: 4, 3, 2, 1, 5 Rationale: The process of generating and selecting interventions occurs in the following order: 1) Review the nursing diagnosis. 2) Review the desired patient outcomes. 3) Identify several interventions or actions. 4) Choose the best interventions for the patient. 5) Individualize standardized interventions to meet the patient's unique needs. Interventions flow from diagnoses and outcomes. Many interventions may be available for a patient problem; select the ones that are best for the patient. If you are using standardized plans of care, protocols, and so forth, you will need to individualize standardized interventions to meet the patient's unique needs.

A nursing diagnostic statement is a three-part statement comprising the diagnostic label, related factors, and the defining characteristics that support the label.

True

Aggregate nursing goals are formulated to address the needs of families, groups, or communities.

True

Implementation refers to the action phase of the nursing process in which nursing care is provided.

True

Nurses manage collaborative problems using nursing and physician-prescribed interventions.

True

The key elements of the nursing plan of care are the nursing diagnosis, client goals, and nursing interventions.

True

Asthma would be a greater threat to oxygenation than hay fever.

True Rationale: Asthma causes edema and spasms of the lower airway, causing breathing to become ineffective in terms of gas exchange. In contrast, hay fever is associated with a runny nose and some edema in the upper airways.

A person with a PCO2 of 28 mm Hg may experience facial numbness, tingling, or both.

True Rationale: Decreased carbon dioxide (an acid) or increased bicarbonate (a base) level makes the body too alkaline, a condition called alkalosis. Alkalosis is usually accompanied with low blood potassium level, causing, numbness, tingling, and muscle cramps (from disturbed function of the skeletal muscles), and muscle spasms (from disturbed function of smooth muscles).

Nursing interventions are performed for the purpose of assessing health status, preventing and treating illness or disease, and promoting health.

True Rationale: Nursing interventions are actions based on clinical judgment and nursing knowledge. They are performed for the purpose of assessing health status, preventing and treating illness or disease, and promoting health.

One definition of outcomes evaluation is "the judgment of the effectiveness of nursing care to meet patient needs."

True Rationale: Outcomes evaluation is the process by which we analyze whether the care delivered for the nursing problem achieved the established goals.

Subcutaneous heparin is one method to prevent the development of a pulmonary embolus in an immobile patient.

True Rationale: Subcutaneous heparin prevents the formation of deep vein thromboses, which can become pulmonary emboli.

When checking a patient to be sure her body is aligned properly, it is desirable to observe the patient from the front, back, and side views.

True Rationale: The front, back, and side views are needed to assess curvatures of the spine, posture, line of gravity, and general balance and symmetry.

Being inquisitive, open-minded, and systematic are traits of critical thinkers.

True Rationale: There are many attributes of a critical thinker. These are just three traits.

Logrolling

Used after spinal surgery

The heart's ability to spontaneously trigger cardiac activity to set minimum heart rate is called ________.

automaticity Rationale: The spontaneous rhythm of the heart is called automaticity.

A medical diagnosis describes a(n) ___________, illness, or injury.

disease and sometimes symptoms such as pain and constipation (which are also nursing diagnoses)

sympathetic

fight or flight The ________ system causes increasing the heart rate.

Parasympathetic

rest and digest The ________ system controls slowing of the heart rate.


Kaugnay na mga set ng pag-aaral

Challenges facing the United States of America in implementing universal coverage

View Set

CSR-Chapter 6 - Ethical Decision Making: Employer Responsibilities & Employee Rights

View Set

Dental Radiography Chapter 10 Quality Assurance in the Dental Office

View Set

Into the Wild Chapter 3-4 Vocabulary

View Set

Med. Term - Chapter 15, The Nervous System

View Set